Sei sulla pagina 1di 38

Surgery Reviewer (FOLD/COVER IF YOU DON’T WANNA SEE THE

ANSWERS YET)
1. A man sustained severe blunt injuries in a car crash twelve hours ago. The activity of this hormone is now A
expected to be decreased in this patient: (Classification-Application; Source-Schwartz’s
a. insulin textbook of Surgery 8th edition: pp 25-29)
b. cortisol
c. epinephrine
d. aldosterone

2. A young woman in debilitated by pruritus and burning sensation from her pre-sternal keloid. The B
recommended initial treatment is: (Classification-Recall; Source-Schwartz’s textbook of
a. topical application of silicone sheets Surgery 8th edition: pp. 241-242)
b. intralesional corticosteroid injection
c. surgical excision
d. low-dose radiation

3. A patient is placed on NPO in preparation for an elective major abdominal surgery. The body’s preferred A
initial fuel source during the fasting state is: (Classification-Recall; Source-Schwartz’s textbook of
a. hepatic glycogen Surgery 8th edition: p. 24)
b. skeletal muscle glycogen
c. muscle protein
d. body fat

4. A 48-year-old woman has prolonged ileus after surgery for an obstructed duodenal ulcer. The problem is D
probably due to: (Classification-Application; Source-Schwartz’s
a. hypochloremia textbook of Surgery 8th edition: pp 48-49)
b. hypocalcemia
c. hypomagnesemia
d. hypokalemia
5. A man with chronic prepyloric ulcer appears weak after repeated bouts of non-bilious vomiting over the past C
three days. Fluid therapy should be started using: (Classification-Application; Source-Schwartz’s
a. Normosol M textbook of Surgery 8th edition: pp 51-52)
b. Lactated Ringer’s Solution
c. Normal Saline Solution
d. Hypertonic Saline Solution

6. A multiply injured patient has persistently low urine output. The oliguria is most likely due to pre-renal B
failure rather than acute tubular necrosis if the tests reveal: (Classification-Application; Source-Schwartz’s
a. low urine specific gravity textbook of Surgery 8th edition: pp 348-349)
b. low urinary excretion of sodium
c. low BUN/creatinine ratio
d. low creatinine clearance

7. A woman admitted to the ICU for severe acute pancreatitis begin to manifest paresthesia of the face and A
extremities, muscle cramps, and a positive Chvostek sign. These are probably due to: (Classification-Application; Source-Schwartz’s
a. hypocalcemia textbook of Surgery 8th edition: p. 50)
b. hypokalemia
c. hypomagnesemia
d. hypophosphatemia

8. Increased nutritional support appears to aggravate tachypnea in a septic patient with respiratory failure. A
Lessening the amount of this substrate may alleviate the problem: (Classification-Recall; Source-Schwartz’s textbook of
a. carbohydrate Surgery 8th edition: pp 28-29)
b. fat emulsion
c. essential amino acids
d. non-essential amino acids

9. Administration of this amino acid is expected to be beneficial to a patient suffering from damage to intestinal D
mucosa due to adjuvant chemotherapy for esophageal cancer. (Classification-Recall; Source-Schwartz’s textbook of
a. Arginine Surgery 8th edition: p. 38)
b. Valine
c. Leucine
d. Glutamine

10. Early supplementation of this vitamin is recommended to promote wound repair in a patient with B
extensive second degree flame burns (Classification-Application; Source-Schwartz’s
a. A textbook of Surgery 8th edition: p. 210)
b. C
c. D
d. E

11. A diabetic 41-year-old man is admitted for necrotizing fasciitis in the perineum. A high dose of this C
antibiotic generally included in the initial antimicrobial therapy because of concern for clostridial pathogens. (Classification-Recall; Source-Schwartz’s textbook of
a. Metronidazole Surgery 8th edition: p.122)
b. Vancomycin
c. Penicillin G
d. Aminoglycoside

12. Transfusion of properly cross-matched blood is begun on a man admitted for massive bleeding from erosive A
gastritis and thirty minutes later, he develops urticaria and fever. This should be administered to the patient. (Classification-Application; Source-Schwartz’s
a. anti-histamine textbook of Surgery 8th edition: pp 79-80)
b. Mannitol
c. Furosemide
d. Sodium Bicarbonate

13. A patient under anticoagulation therapy using warfarin, who is set to undergo surgery for acute B
cholecystitis, has decreased prothrombin concentration. Warfarin can be reversed by parental dose of (Classification – Application; Source Schwartz’s
a. Protamine sulfate Textbook of Surgery (8th edition): pp 73-74)
b. Vitamin K
c. EACA
d. Hydroxyurea
14. Arterial blood gas analysis is performed on a patient just admitted with diagnosis of severe acute A
pancreatitis. This reveals a pH of 7.30 and levels of bicarbonate of pCO2. The most urgent part of management (Classification – Application; Source Schwartz’s
is Textbook of Surgery (8th edition): pp 50-51)
a. Volume resuscitation
b. Intravenous bicarbonate
c. Calcium infusion
d. Mechanical ventilation
15. Seven days after surgery for perforated appendicitis, the primarily closed incision is noted to be D
erythematous, slightly swollen and tender. The appropriate treatment is: (Classification – Application; Source Schwartz’s
a. Local heat therapy Textbook of Surgery (8th edition): pp 119-120)
b. Topical antibiotics
c. New systems antibiotics
d. Incision and drainage
16. A man is brought to the E.R. with blood spurting from a hacking wound is the distal right thigh. He is alert A
and has a systolic BP of 100 mmHg. What is the initial management step? (Classification – Application; Source Schwartz’s
a. Apply direct pressure on the wound with sterile gauze Textbook of Surgery (8th edition): pp 74)
b. Apply digitial pressure on proximal femoral artery
c. Apply a thigh tourniquet above the wound
d. Open the wound and clamp the bleeders

17. A young man presents to the E.R with a stab wound in the left chest. Examination reveals subcutaneous B
emphysema and absent breath sounds on the left chest; the trachea is shifted to the right. What is the (Classification – Application; Source Schwartz’s
probable diagnosis Textbook of Surgery (8th edition): pp 131)
a. Massive hemothorax
b. Tension pneumothorax
c. Cardiac tamponade
d. Flail chest
18. A 58-year old woman NSAID therapy for arthritis has a 6 hr history of epigastric pain that has progressively A
become severe. Her abdomen is distended, rigid and diffusely tender. This diagnostic test should be done first: (Classification – Application; Source Schwartz’s
a. Upright chest X-ray Textbook of Surgery (8th edition): pp 959-960)
b. Esophagogastroduodenoscopy
c. Abdominal ultrasound
d. Abdominal CT scan

19. After aggressive fluid therapy, a trauma victim shows a systolic BP of 110 mmHg, cold extremities, rapid but A
strong peripheral pulse, and a central venous pressure of 12 cmH20. The patient has: (Classification – Application; Source Schwartz’s
a. Increased systemic vascular resistance Textbook of Surgery (8th edition): pp 132)
b. Decreased stroke volume index
c. Decreased cardiac index
d. Excessive cardiac preload

20. An elderly man admitted with a diagnosis of sigmod volvulus has a markedly distended, non-tender A
abdomen and hyperactive bowel sounds. The initial treatment is: (Classification – Application; Source Schwartz’s
a. Endoscopic detorsion Textbook of Surgery (8th edition): pp 1098)
b. Detorsion via laparotomy
c. Decompressing transverse loop colostomy
d. Resection of involved segment of colon
21. Pancreatic necrosis is suspected in a 49-year-ol woman who has not improved despite 3 days of intensive C
care for acute pancreatitis. The diagnosis is best established though: (Classification – Application; Source Schwartz’s
a. C-reactive measurement Textbook of Surgery (8th edition): pp 1234-1238)
b. Abdominal ultrasound
c. Contrast-enhanced CT scan
d. CT-guided percutaneous biopsy
22. A chronically constipated 67-year-old woman presents with acute left lower quadrant (LLQ) abdominal pain, B
LLQ direct and rebound tenderness, and fever. The appropriate diagnostic examination is: (Classification – Application; Source Schwartz’s
a. Transvaginal ultrasound Textbook of Surgery (8th edition): pp 1082-1083)
b. Abdominal CT scan
c. Proctosigmoidoscopy
d. Barium enema

23. After an elective hemicolectomy for colon cancer, a 78-year-old man remains hypotensive and tachycardic. B
He has distended neck veins, cold skin, oliguria, and elevated central venous pressure. He apparently is (Classification – Application; Source Schwartz’s
suffering from this type of shock: Textbook of Surgery (8th edition): pp 95-100)
a. Hypovolemic
b. Cardiogenic
c. Septic
d. Neurogenic

24. For the past 6 weeks, a 67-year-old has been asymptomatic except for constipation after a course of C
antibiotic therapy for left lower quadrant abdominal pain. He should undergo: (Classification – Application; Source Schwartz’s
a. CEA determination Textbook of Surgery (8th edition): pp 1082-1083)
b. Barium enema
c. Colonoscopy
d. Abdominal CT scan

25. A 32-year-olf man with a week’s history of fever presently has right lower quadrant abdominal pain and A
tenderness and bloody diarrhea. The most probable diagnosis is: (Classification – Application; Source – Schwartz’s
a. typhoid ileitis Textbook of Surgery (8th edition): pp. 1049-1050)
b. amebic colitis
c. ileocecal TB
d. intussusception

26. A 75 year old woman in the ICU after undergoing cholecystectomy for acute cholecystitis is hypotensive C
and tachycardic. Pulmonary capillary wedge pressure (PSCP) is elevated to 18mmHg, and cardiac output is (Classification – Recall; Source – Schwartz’s Textbook
3L/min. Hershock is best described as which of the following? of Surgery (8th edition): pp.)
a. hypovolemic shock
b. septic shock
c. cardiogenic shock
d. anaphylactic shock

27. A 19 year old man is brought to the emergency department with a stab wound at the base of the neck C
(zone 1). The most important concern for the patients with such injuries is which of the following? (Classification – Recall; Source – Schwartz’s Textbook
a. upper extremity ischemia of Surgery (8th edition): pp.)
b. cerebral infarction
c. exsanguinating hemorrhage
d. mediastinitis

28. In septic shock, which of the following is true? C


a. The mortality rate is between 10 and 20% (Classification – Recall; Source – Schwartz’s Textbook
b. Gram negative organisms are involved exclusively of Surgery (8th edition): pp.)
c. The majority of patients are elderly
d. The most common source of infection is the alimentary tract.

29. The injury most often missed by selective nonoperative management of abdominal stab wounds is to which D
of the following? (Classification – Recall; Source – Schwartz’s Textbook
a. colon of Surgery (8th edition): pp.)
b. spleen
c. ureter
d. diaphragm

30. A 63-year-old male tobacco smoker has a 1.5 cm non-healing ulcer in his lower lip. A nodular lesion is A
palpable deep to the ulcer. The most likely diagnosis is: (Classification – Application; Source – Schwartz’s
a. squamous cell carcinoma Textbook of Surgery (8th edition): pp. 518)
b. keratoacanthoma
c. malignant fibrous histiocytoma
d. verrucous carcinoma

31. A 43-year-old-man presents with nasal obstruction and occasional epistaxis. Imaging studies combined C
with endoscopic bipsy have led to the diagnosis of nasopharyngeal cancer. The standard treatment is: (Classification – Application; Source – Schwartz’s
a. intracavitary radiation Textbook of Surgery (8th edition): pp. 1040-1050)
b. external beam radiation
c. combined chemotherapy and radiation
d. surgical resection
32. Biopsy of a slow growing infraauricular mass reveals a benign pleomorphic adenoma that is confined to the C
superficial lobe of the parotid gland. The standard treatment is: (Classification – Recall; Source – Schwartz’s Textbook
a. tumor enucleation of Surgery (8th edition): pp 538-540)
b. tumor excision with 2mm margin
c. superficial parotidectomy
d. surgical resection

33. A 58 year old woman undergoes excision biopsy of a tumor in the left posterior triangle of her neck.
Histology suggests that this is a metastatic cancer. What is the most likely site of the primary tumor? (Classification – Recall; Source – Schwartz’s Textbook
a. Ovary of Surgery (8th edition): pp)
b. Adrenal Gland
c. Kidney
d. piriform fossa

34. A 59 year old woman has discomfort in the posterior part of her tongue. A biopsy confirms that the lesion is B
a carcinoma. What is true in carcinoma of the posterior third of the tongue? (Classification – Recall; Source – Schwartz’s Textbook
a. lymphoid tissue is absent of Surgery (8th edition): pp)
b. lymphoid gland spread is often encountered
c. there is an excellent prognosis
d. the tissue is well differentiated

35. A 60 year old man has a 3 cm nodular lesion with central ulceration in his left cheek. Section biopsy proves D
this to be basal cell carcinoma. This is best managed with: (Classification – Application; Source – Schwartz’s
a. external beam radiation Textbook of Surgery (8th edition): pp 439-440)
b. topical 5-fluouracil
c. electrodessication
d. excision with 2-4mm margin

36. This is the most common aggressive primary malignant bone tumor in adolescent and occurs in B
methaphyseal area of long bones with high incidence of pulmonary metastases: (Classification – Recall; Source – Schwartz’s Textbook
a. Chrondrosarcoma of Surgery (8th edition): pp)
b. Osteosarcoma
c. Fibrosarcoma
d. Adamantinoma

37. A 65-year-old farmer presents with a 1.5-cm ulcerated lesion on the middle third of his lower lip. The lesion A
has been present for 4 months and is not painful. No lymph nodes are palpable in the patient’s neck. The most (Classification – Application; Source – Principles of
likely diagnosis is; Surgery by Schwartz, Spencer, Shires 7th edition Volp
a. Squamous cell carcinoma pp 521-522
b. Basal cell carcinoma
c. Herpes simplex
d. Keratoacanthoma

38.A 65-year-old patient who spends summer in Baguio City presents with a painless, ulcerated lesion on the C
right cheek. The lesion has been present for one year. Physical examination of the patient’s neck reveals no (Classification – Application; Source – Principles of
lymph node enlargement. The most likely diagnosis is: Surgery by Schwartz, Spencer, Shires 7th edition Vol
a. pyogenic granuloma 1, p 522)
b. melanoma
c. basal cell carcinoma
d. squamous cell carcinoma

39. A 60-year-old woman has a 10 day history of cough and fever. Imaging studies show multiple fluid B
loculations in the right chest cavity with an estimated volume of 500 ml. Thoracentesis draws purulent (Classification – Application; Source – Principles of
material. The best treatment strategy now is: Surgery by Schwartz, Spencer, Shires 7th edition Vol
a. pigtail catheter drainage of the empyema 1, pp 601-602)
b. video assisted thorascopic drainage with deloculation
c. insertion of multiple thoracostomy tubes
d. early thoracotomy and drainage

40. A 50-year-old woman presents with a 3 cm solitary pulmonary nodule in the left upper lobe. She has a C
history of total thyroidectomy for papillary cancer at the age of 38 years. The main consideration is: (Classification – Application; Source – Schwartz’s
a. TB granuloma Textbook of Surgery (8th edition): pp 556-557)
b. hamartoma
c. primary lung cancer
d. metastatic thyroid cancer

41.A 45-year-old woman presents with weakness towards the afternoon, shortness of breath, and ptosis. CT D
scan reveals an anterior mediastinal mass. The most likely diagnosis is: (Classification – Application; Source – Schwartz’s
a. intrathoracic goiter Textbook of Surgery (8th edition): pp 593-594)
b. non-Hodgkin’s lymphoma
c. non-seminomatous germ cell tumor
d. thymoma
42. A man with malignant pleural effusion in association with an inoperable lung cancer may benefit from the A
instillation of sclerosing agent into the pleural cavity, once the lungs are almost fully expanded. An accepted (Classification – Application; Source – Schwartz’s
sclerosant is: Textbook of Surgery (8th edition): pp 600-601)
a. talc
b. gentamicin
c. penicillin G
d. Vitamin

43. A 20-year-old woman consults for a 2-cm mass in her left breast. The mass is movable, non-tender, and has A
a rubbery consistency and smooth borders. The probable diagnosis is: (Classification – Application; Source – Schwartz’s
a. fibroadenoma Textbook of Surgery (8th edition): pp 463-464)
b. fibrocystic disease
c. carcinoma
d. cytosarcoma phyllodes

44. A 16-year-old girl is bothered by cyclical premenstrual pain her breasts. She has 0.5-1.0 cm nodularities in A
the upper outer quadrant of both breasts. What management advice should be given to her? (Classification – Application; Source – Schwartz’s
a. Observation Textbook of Surgery (8th edition): pp 465-466)
b. breast ultrasound
c. mammography
d. aspiration biopsy

45. An asymptomatic 40-year-old woman with jno palpable breast mass undergoes mammography. This reveals D
clustered microcalcifications in the lower outer quadrant of her left breast. The probable diagnosis is: (Classification – Application; Source – Schwartz’s
a. florid hyperplasia Textbook of Surgery (8th edition): pp 472-473)D
b. sclerosing adenosis
c. atypical ductal hyperplasia
d. ductal carcinoma in situ

46. A 45-year-old woman with “lumpy” breasts undergoes mammography. Mammographic findings are B
suggestive of high grade ductal carcinoma in situ confined to the upper outer quadrant of her right breast. How (Classification – Application; Source – Schwartz’s
should she be managed? Textbook of Surgery (8th edition): p 481)
a. lumpectomy alone
b. lumpectomy and radiotheraphy
c. mastectomy
d. mastectomy and axillary node sampling

47. A 45-year-old woman has a recent onset bloody nipple discharge from her right breast. No palpable breast B
mass is noted. What diagnostic test is indicated? (Classification – Application; Source – Schwartz’s
a. breast ultrasound Textbook of Surgery (8th edition): p 493)
b. mammography
c. ductography
d. cytology of discharge

48. A premenopausal woman undergoes modified radical mastectomy for a 3-cm breast cancer. No axillary D
node and distant metastases are detected. Test for this biomarker is currently recommended to facilitate the
selection of adjuvant chemotheraphy.
a. c-fos
b. c-myc
c. p53
d. AHER2/neu

49. A 63-year old man with chronic atrial fibrillation has sudden onset of pain, weakness and paresthesia in his A
left leg, which appears cool, cyanotic and without femoral and distal pulses. The right leg has normal pulses.
The most likely diagnosis is:
a. arterial embolism
b. aortoillac thrombosis
c. Buerger’s disease
d. Raynaud’s disease

50. Coronary angiogram reveals a triple-vessel disease in a diabetic 55-year-old man presenting with unstable C
angina. The recommended treatment is:
a. catherter-directed thrombolysis
b. balloon angioplasty
c. coronary artery bypass grafting
d. transmyocardial revascularization

51. A 41-year old male chronic smoker has a 2-year history of bilateral foot claudication. He now has ulceration A
of the tip of the right 3rd toe and left 2nd and 4th toes. Popliteal pulses are palpable but the posterior tibial and
dorsalispedis pulses are bilaterally absent. The most important step in management is:
a. cessation of smoking
b. infusion of prostaglandin
c. administration of indomethacin
d. immediate surgical correction of PDA

53. A 3-year-old girl presents with cyanotic spells that have increased in severity and frequency since birth. X- B
ray shows a boot-shaped heart, ECG reveals right ventricular hypertrophy. The most probable diagnosis is:
a. Truncusarteriosis
b. tetralogy of Fallot
c. transposition of the great arteries
d. Ebstein’s anomaly

54. A 54-year-old man presents with hematemesis after a bout of vomiting and retching. Endoscopy shows A
linear mucosal tears at the gastroesophageal junction.The diagnosis is:
a. Mallory-Weiss syndrome
b. Boerhaave’s syndrome
c. Menether’s syndrome
d. Dieulafoy’s syndrome

55. After truncalvagotomy and antrectomy with gastrojejunostomy for an obstructed duodenal ulcer, the D
patient complains of diaphoresis, weakness and abdominal discomfort followed by diarrhea several minutes
after meals. The symptoms are suggestive of:
a. post vagotomy diarrhea
b. bile reflux gastritis
c. afferent loop syndrome
d. dumping syndrome

56. A woman undergoes total gastrectomy for a huge proximal gastric carcinoma. To protect her from C
developing anemia, she must be given a regular parenteral dose of:
a. folic acid
b. ferrous sulfate
c. vitamin B12
d. transferrin

57. A few weeks after recovering from severe pancreatitis, a 34-year-old man has developed a pancreatic B
pseudocyst with mature wall that is pressing on his stomach. Aspiration fails to relieve the pressure symptoms.
The indicated treatment is:
a. administration of somatostatin
b. internal drainage (cystogastrostomy)
c. external drainage
d. excision of pseudocyst

58. ERCP reveals a penampullary cancer in a 64-year-old man admitted for jaundice. No metastasis is detected. A
No co-morbid conditionas are identified. How should he be managed?
a. radical excision of the head of pancreas and duodenum
b. local excision and adjuvant chemotheraphy
c. external beam radiation
d. stenting and chemotheraphy

59. A 60-year-old man with no previous operation has a 5-day history of inability to pass flatus or feces, A
cramping abdominal pain, and progressive abdominal distention. The principal diagnostic consideration is: (Classification-Application; Source-Schwartz’s
a. midgut volvulus textbook of Surgery 8th edition: pp 25-29)
b. sigmoid volvulus
c. colorectal cancer
d. ileocecal tuberculosis

60. A man who has an obstructed rectosigmoid cancer in association with a competent ileocecal valve is liable D
to develop perforation of: (Classification-Application; Source-Schwartz’s
a. sigmoid colon textbook of Surgery 8th edition: pp 1089-1090)
b. splenic flexure
c. hepatic flexure
d. cecum

61. The presence of tenesmus, decrease in calibre of stools and occasional bloody-mucoid diarrhea in a 67- A
year-old man is suggestive of: (Classification-Application; Source-Schwartz’s
a. rectal carcinoma textbook of Surgery 8th edition: pp)
b. amoebic colitis
c. intussusceptions
d. ileocecal tuberculosis

62. Diagnostic tests show an ileocolic intussusceptions in a 38-year-old man, whoo has no signs of
strangulation. What is recommended for this patient? (Classification-Application; Source-Schwartz’s
a. expectant treatment textbook of Surgery 8th edition: pp)
b. endoscopic decompression
c. hydrostatic reduction
d. exploratory laparotomy
63. A 63-year old man is presently asymptomatic after medical treatment for LLQ pain and tenderness, He B
should undergo: (Classification-Application; Source-Schwartz’s
a. CEA determination textbook of Surgery 8th edition: pp)
b. Colonoscopy
c. Barium enema
d. Abdominal CT scan

64. A 56-year-old woman taking NSAIDs for chronic arthritis experienced severe epigastric pain 6 hours ago.
Her abdomen is now diffusely tender with board-like rigidity. The appropriate initial diagnostic test is: (Classification-Application; Source-Schwartz’s
a. chest upright xray textbook of Surgery 8th edition: pp)
b. plain abdominal supine xray
c. abdominal ultrasound
d. abdominal CT scan

65. Despite 3 months of medical treatment for posterior midline anal fissure, the man’s fissure-related anal D
painduring and after defecation has become excruciating. The appropriate treatment is: (Classification-Application; Source-Schwartz’s
a. excision of fissure textbook of Surgery 8th edition: pp 1103-1104)
b. botulinum toxin injection
c. anal divulsion
d. internal sphincterotomy

66. During surgery for a large indirect inguinal hernia, the cecum is noted to form part of the wall of the hernia A
sac. The patient has this type of hernia: (Classification-Application; Source-Schwartz’s
a. sliding textbook of Surgery 8th edition: 1358)
b. Richter’s
c. interstitial
d. Spigelian

67. A previously asymptomatic 45-year-old obese woman consults for a non-tender swelling below her right B
inguinal ligament just lateral to the pubic tubercle. A warranted diagnostic procedure for this patient is: (Classification-Application; Source-Schwartz’s
a. fine needle aspiration textbook of Surgery 8th edition: pp 1364-1366)
b. ultrasound
c. MRI
d. herniography

68. A 54 year old male taking anti-coagulant for the past 4 weeks developed 3 day history of abdominal pain. B
On physical examination, there was a palpable mass at the supraumbilical area with direct tenderness. (Classification-Application; Source-Schwartz’s
Fothergill sign was positive. What is your impression? textbook of Surgery 8th edition: pp )
a. desmoid tumor
b. rectus sheath hematoma
c. omental torsion
d. omental infarction

1. A 65 year-old male who underwent a colon resection of carcinoma 2 years prior to consult is found to have 3 D
solid nodules approximately 2 cms each in the right and left lobes of the liver. There is no extrahepatic disease (Classification-Application; Source-Schwartz’s
detected. The preferred treatment is: textbook of Surgery 8th edition: pp)
a. systemic chemotherapy
b. radiofrequency ablation
c. microwave coagulation therapy
d. surgical resection of metastatic nodule

2. A 35 year old female with a prolonged intake of contraceptive pills develop right upper quadrant pains. B
UItrasound shows an isodense 5cm mass in the right lobe of the liver. Sulfur-colloid scan done showed a cold (Classification-Application; Source-Schwartz’s
lesion. The patient should undergo/; textbook of Surgery 8th edition: pp 1089-1090)
a. conservative treatment
b. resection
c. enucleation
d. liver transplantation

69. A 12 year old female complain of 1 day history of epigastric pain which shifted to the right lower quadrant B
associated with anorexia and vomiting. Physical examination revealed direct and rebound tenderness on the (Classification – Application; Source – Schwartz’s
right lower quadrant. With the preoperative diagnosis of acute appendicitis, she was operated on. However, Textbook of Surgery (8th edition): pp.)
intraoperatively there was blood tinged peritoneal fluid and appendix was noted to be normal. The most
probable cause of the patient condition is:
a. mesenteric cyst
b. omental torsion
c. ruptured rectus sheath Hematoma
d. mesenteric adenitis

70. A 45-year-old is discovered to have solitary 2-cm form nodule in his right thyroid lobe. No associated D
symptoms are noted; family history is negative for thyroid cancer. Diagnostic work-up should start with: (Classification – Application; Source – Schwartz’s
a. thyroid function tests Textbook of Surgery (8th edition): pp 1413-1416)
b. thyroid ultrasound
c. thyroid scan
d. fine needle aspiration biopsy

71. A 59-year-old man presents with 4-cm firm mass in the left lobe of the thyroid associated with a form B
occipital mass that is 6cm in its widest diameter, which he has neglected for the past 9 months. Family history (Classification – Application; Source – Schwartz’s
is negative for thyroid malignancy. He probably has this type of thyroid cancer: Textbook of Surgery (8th edition): pp1420-1421)
a. Papillary
b. Follicular
c. medullary
d. anaplastic

72. A 49-year-old woman complains of central obesity with moonlike fascies and hirsutism. Her ACTH is D
markedly elevated which is suppressed by large dose of dexamethasone. The probable pathology is: (Classification – Application; Source – Schwartz’s
a. adrenal adenoma Textbook of Surgery (8th edition): pp 1455-1458)
b. adrenal carcinoma
c. pheochromocytoma
d. pituitary adenoma

73. A 42-year-old man presents with headache, palpitations, and diaphoresis. His urinary metanephrines are A
elevated. Imaging studies reveal a 3-cm mass in his right adrenal gland. The treatment of choice is: (Classification – Application; Source – Schwartz’s
a. Adrenalectomy Textbook of Surgery (8th edition): pp 1460-1461)
b. Radiotherapy
c. tumor embolization
d. chemoradiation

74. A 67-year-old man is bleeding excessively following a transurethral prostatectomy. The bleeding is assesses D
to be due to local plasminogen activation leading to increased fibrinolysis on the raw wound surface. (Classification – Application; Source – Schwartz’s
Administration of this drug may be beneficial: Textbook of Surgery (8th edition): p 82)
a. vitamin K
b. heparin
c. warfarin
d. EACA

75. A 60-year-old man presents with slight urinary urgency, frequency, and a decrease in the force of A
micturition. Digital rectal examination detects no mass in the prostrate. The indicated diagnostic test is: (Classification – Application; Source – Schwartz’s
a. prostatic acid phosphatase Textbook of Surgery (8th edition): pp 1535-1537)
b. prostate specific antigen
c. pelvic ultrasound
d. needle biopsy of prostrate

76. A 35-year-old man presents with right flank pain and microscopic hematuria. Intravenous pyelography A
reveals a 4 mm radiolucent stone in the distal right ureter. The recommended treatment is: (Classification – Application; Source – Schwartz’s
a. alkalinziation of the urine Textbook of Surgery (8th edition): pp 1546-1549)
b. extracorporeal shock wave lithotripsy
c. stone extraction via ureteroscope
d. open ureterolithomy

77. A 55-year-old man is admitted for gross hematuria associated with left flank plain and anemia. CT scan A
detects a solid, enhancing mass in the left kidney. The likely diagnosis is: (Classification – Application; Source – Schwartz’s
a. renal cell carcinoma Textbook of Surgery (8th edition): pp 1531-1532)
b. renal tuberculosis
c. renal sarcoma
d. renal oncocytoma

78. A 23-year-old woman sustained blunt head trauma to her right frontoparietal area in a car crash, which C
caused him to be unconscious for several minutes. Upon arrival at the ER, she is alert and has no neurologic (Classification – Application; Source – Schwartz’s
deficit. X-ray shows a linear on-depressed right frontoparietal fracture. An hour after admission, vomiting Textbook of Surgery (8th edition): p 1619)
followed by progressive neurologic deterioration are noted.
a. diffuse axonal injury
b. cerebral contusion
c. acute epidural hematoma
d. acute subdural hematoma

79. A 54-year-old woman with no previous neurologic disorder has a new onset seizure without associated C
gross neurologic deficit. She was treated for breast cancer 10 years ago. If brain metastasis is the main (Classification – Application; Source – Schwartz’s
consideration, this is best confirmed by means of: Textbook of Surgery (8th edition): pp 1633)
a. A. Electroencephalography
b. B. CT scan of the brain with intravenous contrast
c. C. MRI of the brain with intravenous contrast
d. D. Doppler ultrasound
80. A 55-year-old man with a history of lung cancer presents with frequent severe headache, occasional C
vomiting, and weakness of the right upper extremity. What is the recommended treatment in addition to (Classification – Application; Source – Schwartz’s
corticosteroids? Textbook of Surgery (8th edition): p 1633)
a. A. intrathecal chemotherapy
b. B. whole-brain radiotherapy
c. C. stereotactic radiosurgery (gamma knife)
d. D. resection of lesion causing arm weakness

81. A young man just admitted for blunt injuries from a car accident is stuporous. CT scan shows cerebral B (Classification – Recall; Source – Schwartz’s
contusion. What should be avoided as it can cause secondary brain injury to this patient through its deleterious Textbook of Surgery (8th edition): pp. 1613-1614)
effect on intracranial pressure?
A. elevation of head
B. respiratory acidosis
C. hypovolemia even of mild degree
D. sedation agitated patient

82. As a basketball player falls hard on his extended right arm, he feels immediate severe pain with subsequent B (Classification – Application; Source - Schwartz’s
inability to move his right arm. Swelling and tenderness about the shoulder with loss of deltoid contour are Textbook of Surgery (8th edition): pp. 1706-1707)
noted. The likely diagnosis is:
A. deltoid muscle rupture
B. shoulder dislocation
C. fracture of proximal humerus
D. brachial plexus injury

83. A 10-week-old baby with cleft lip and palate has undergone a successful cleft lip repair. It is recommended B (Classification – Recall; Source – Schwartz’s
that cleft palate repair be done before the baby reaches this age: Textbook of Surgery (8th edition): pp. 1797-1799)
A. 6 months
B. 12 months
C. 2 years
D. 4 years

84. A bedridden 61-year-old woman has a significant neurologic recovery from a previous stroke. She has an C (Classification – Recall; Source – Schwartz’s
infected deep pressure ulcer in her sacrum. Once the infection has been controlled by debridement and Textbook of Surgery (8th edition): pp. 1825-1826)
supportive treatment, the pressure ulcer can be managed with:
A. skin graft
B. rotation skin flap
C. gluteus maximus flap
D. microsurgery free flap

85. A small thick antecubital scar from a 2nd degree flame burn is constricting movement. To relieve the A (Classification – Recall; Source – Schwartz’s
constriction, the appropriate procedure to refashion the scar is: Textbook of Surgery (8th edition): pp. 1792-1793)
A. Z-plasty
B. advancement flap
C. scar excision and skin graft
D. deltopectoral flap

86. Resection of a sarcoma results in a significant lateral chest wall defect. To minimize pulmonary dysfunction, D (Classification – Recall; Source – Schwartz’s
chest wall reconstruction may utilize: Textbook of Surgery (8th edition): pp. 1823-1824)
A. rectus abdominis muscle flap
B. omental flap
C. tensor fascia lata graft
D. Marlex mesh

87. A 4-week old baby boy presents with frequent non-bilious vomiting. During feeding peristaltic waves are A (Classification – Application; Source – Schwartz’s
seen in the upper abdomen in association with a palpable mass in the right upper quadrant. The diagnosis is: Textbook of Surgery (8th edition): pp. 1486-1487)
A. hypertrophic pyloric stenosis
B. duodenal atresia
C. jejuna atresia
D. volvulus neonatorum

88. A newborn infant has excessive drooling followed by coughing immediately after feeding. The main A (Classification – Application; Source – Schwartz’s
diagnostic consideration is esophageal atresia. This can be readily confirmed by: Textbook of Surgery (8th edition): pp. 1481-1482)
A. inability to pass orogastric tube into the stomach
B. water-soluble contrast study (esophagram)
C. transesophageal ultrasound
D. plain abdominal radiograph

89. A diagnosis of intussusceptions is made in an infant with paroxysms of crampy abdominal pain and C (Classification – Recall; Source – Schwartz’s
vomiting. The infant has no peritoneal signs and remains hemodynamically stable. The initial treatment of Textbook of Surgery (8th edition): pp. 1493-1494)
choice is:
A. nasogastric decompression
B. endoscopic reduction
C. pneumatic reduction
D. hydrostatic reduction with barium

90. A 3-day old infant who has not passed meconium presents with abdominal distention and bilious vomiting. D (Classification – Application; Source – Schwartz’s
The most likely diagnosis is: Textbook of Surgery (8th edition): pp. 1496-1497)
A. duodenal atresia
B. malrotation
C. midgut volvulus
D. Hirschsprung’s disease

91. A 3-week-old infant is noted to have jaundice in association with acholic stools shortly after birth. A B (Classification – Application; Source – Schwartz’s
radionuclide scan demonstrates presence of radioisotope in the intestine. This finding rules out the diagnosis of Textbook of Surgery (8th edition): pp. 1499-1500)
:
A. neonatal hepatitis
B. biliary atresia
C. inspissated bile syndrome
D. choledochal cyst

92. A 67 y/o man has an intraabdominal abscess caused by perforated sigmoid diverticulitis. The most common B Schwartz’s: surgical infection p.121
isolated microorganism is _?
A. Escherichia coli
B. Bacteroides
C. K. pneumonia
D.Staphylococcus epidermides

93. One of the most common complications of central venous access is ____. B Schwartz’s Complication in Surgery p. 337
A. hemorrhage
B. pneumothorax
C. wound infection
D. air embolus

94. Early postoperative small bowel obstruction is a rare finding and occurs less than 1% of the time. When it B Schwartz’s: Surgical Complications p. 347
does, the most common cause is ____.
A. internal hernias
B. post-operative adhesions
C. infections or abscess
D. technical errors

95. Which of the following statements is true concerning corrosive injury to the esophagus? C Schwartz’s: Esophagus p.909
A. Acid ingestion is not injurious to the stomach because of its non-acidic pH.
B. Ingested caustic agents rapidly pass through the esophagus and stomach into the small intestines.
C. Alkaline injury is more destructive than acid injury.
D. Acid injury is more destructive than alkaline injury.

96. A 27 year old female was brought to the ER with history of suicidal ingestion of household muriatic acid. B Schwartz’s: Esophagus p.910
This patient is best managed by ----
A. Give half strength vinegar, lemon juice or orange juice
B. Start with milk, egg white or antacids
C. Give emetics to induce vomiting
D. Sodium bicarbonate is one of the treatment option

97. Which of the following statements is true regarding the arterial blood supply of the stomach? B Schwartz’s: Stomach pp.935-937
A. The right gastric artery, a branch of the superior mesenteric artery supplies the gastric antrum
B. Because of the rich intramural collateral vessels, gastric viability may be preserved after ligation of at least
two of the four named gastric arteries.
C. The largest artery supplying the stomach is the right gastric artery.
D. The left gastroepiploic artery is a branch of the celiac trunk.

98. At a cellular leve, the major stimulants of acid secretion by the gastric parietal cell are the following, except: B Schwartz’s: Stomach p. 943
A. histamine
B. prostaglandinE2
C. acetylcholine
D. gastrin

99. A patient with gastric adenocarcinoma underwent subtotal gastrectomy. Histopath revealed tumor B Schwartz’s: Stomach p. 970
penetrating the serosa, regional lymph nodes are not involved and distant metastatic lesions are not detected.
What is the correct tumor stage on TNM staging?
A. Stage I
B. Stage II
C. Stage III-A
D. Stage III-B

100. A 5-month old baby boy was seen at the ER with complaints of non-bilous vomiting that became C Schwartz’s: Pediatric Surgery p. 1486
increasingly projectile over several days to weeks and cannot tolerate liquid intake. Abdominal examination
revealed palpation of typical “olives” in the right upper quadrant and cannot tolerate liquid intake. Abdominal
examination revealed palpation of typical “olives” in the right upper quadrant with visible gastric waves. What
is the correct diagnosis?
A. meconium ileus
B. Hirschprung’s disease
C. Hypertrophic pyloric stenosis
D. Intussusception

101. Which of the following is the most common malignant liver tumor in childhood? B Schwartz’s: Pediatric Surgery p. 1512
A. Hemangioma
B. Hepatoblastoma
C. Hepatocellular carcinoma
D. Mesenchymal hamartoma

102. What is the appropriate treatment for an incidental 3cm liver hemangioma found on abdominal CT scan of A Schwartz’s: pp. 1162-1163
37 year old woman?
A. No treatment
B. Cryoablation
C. Selective embolization of left hepatic artery
D. Left hepatic lobectomy

103. A 35 y/o male presents with 1 year history of early satiety and right upper abdominal discomfort. CT scan C Schwartz’s: p. 1159
shows a 6x8 cm cystic lesion in the right lobe of the liver. Ultrasound of cyst shows no internal echoes. The
appropriate management is:
A. cryoablation of cyst
B. right hepatic lobectomy
C. unroofing of the cyst
D. percutaneous aspiration of cyst

104. An imaginary line that divides the liver into right and left lobe that runs from the inferior vena cava to the C Schwartz’s: p. 1140
tip of the gallbladder fossa:
A. Catlie line
B. Calot’s line
C. Cantlie’s line
D. Charcot’s line

105. The diagnostic tool important in evaluating liver injury in a stable blunt trauma patient is: C Schwartz’s: p. 142
a. Ultrasound c. CT – Scan
b. Abdominal X-ray d. Peritoneal Lavage

106. The local factor affecting wound healing C Schwartz’s: p. 235


a. Age c. Low oxygen tension
b. Smoking d. Trauma

107. The characteristics of keloid scars: C Schwartz’s: p. 240


a. Arise above the skin level and stay in the confine of the original wound
b. Develop within 4 weeks after trauma
c. Extend beyond the border of the original wound and rarely regresses spontaneously
d. Occur in areas of flexion and extension

108. A 55 y/o male diagnosed to have chronic pancreatitis, developed epigastric mass measuring about 6 cm C Schwartz’s: p. 1256
accompanied by pain and fullness. The diagnosis is:
a. Gastric Carcinoma c. Pseudocyst of the pancreas
b. Pancreatic Carcinoma d. Acute pancreatitis

109. Type of benign solitary pancreatic neoplasm consisting of symptomatic fasting hypoglycemia and profound A Schwartz’s: p. 1275
syncopal episodes:
a. Insulinoma b. Glaucoma c. Gastrinoma d. VIPoma

110. The most common indication for splenectomy: C Schwartz’s: p. 1301


a. Staging of Hodgkin’s disease c. Trauma to spleen
b. Hereditary spherocytosis d. Thalassemia

111. The best time to repair of cleft lip is: B Schwartz’s: p. 1797
a. 10 months b. 10 weeks c. 6 months 16 weeks

112. A 36 y/o female complain of an acute pain at the lateral aspect of her breast. A tender firm cord is found B Schwartz’s: p. 463
on the same site on physical examination. It was diagnosed as a Mondors disease. This lesion is best managed
with:
a. Radiotherapy
b. Anti-inflammatory medications, warm compress and rest of ipsilateral extremity
c. Immediate excision of the lesion is needed
d. Total mastectomy
113. Breast lesion that can be treated with close observation with or without tamoxifen: B Schwartz’s: p. 481
a. Early invasive breast carcinoma c. Ductal carcinoma in situ
b. Lobular carcinoma in situ d. Inflammatory carcinoma of the breast

114. The best screening imaging technique for breast cancer is: D Schwartz’s: p. 476-477
a. Magnetic resonance imaging c. Ductography
b. Ultrasonography d. Mammography

115. A 46 y/o female who underwent a total thyroidectomy for stage II papillary carcinoma developed 3 weeks C Schwartz’s: p. 1429-1448
later with Chvostek’s and Trouseau’s sign. This could be due to:
a. Recurrence of papillary thyroid carcinoma c. Hypoparathyroidism
b. Injury to the cervical sympathetic trunk d. injury to the recurrent laryngeal nerve

116. This is considered as a single most important test in the evaluation of patients with thyroid masses C Schwartz’s: p. 1415
a. Thyroid ultrasound c. Fine needle aspiration
b. CT and MRI d. Thyroid Scanning

117. Lymph node in the posterior triangle of the neck is within what level: C Schwartz’s: p. 534
a. Level II b. Level IV c. Level V d. Level VI

118. This form of shock has a low blood pressure, low urine output but has a elevated central venous pressure: B Schwartz’s: p. 95-102
a. Vasodilitory shock c. Neurogenic shock
b. Obstructive shock d. Traumatic shock

119. 55 y/o male who has been in the ward for the past 4 days and is receiving IV fluid of D5W is complaining C Schwartz’s: p.48
on his 5th hospital day of headache, nausea, vomiting, anorexia, body malaise and watery diarrhea. Give the
most probable electrolyte imbalance the patient might have:
A. Hypernatremia B. Hyperkalemia C. Hyponatremia D. Hypokalemia

120. the most important part of the treatment of severe metabolic acidosis among trauma patient is: C Schwartz’s: p.50-51
A. Correct abnormality with exogenous bicarbonate C. Restore perfusion with volume resuscitation
B. High Oxygenation D. Give colloid solution

121. the most common primary bone malignancy: A Schwartz’s: p.1665


A. Osteosarcoma B. Ewing’s Sarcoma C. Ostoblastoma D. Chondrosarcoma

122. The most potent stimulant for aldosterone release is: A Schwartz’s: p.9
A. ACTH B. rennin C. angiotensin I D. angiotensin II

123. the most biologically active and potent vasoconstrictor is: B Schwartz’s: p.23
A. angiotensin II B. endothelins C. epinephrine D. Serotonin

124. In trauma patients significant reduction in infectious complications were noted in patients given early C Schwartz’s: p.431
enteral nutrition as compared to those given who are unfed or given parenteral nutrition except for what type
of trauma patients?
A. Renal Trauma B. Long bone fractures C. Head trauma D. Liver trauma

125. Branched chain amino acids are used in parenteral nutrition for what specific type of patients? D Schwartz’s: p.33
A. renal failure patients C. pulmonary failure patients
B. diabetic patients D. hepatic failure patients

126. In what type of parenteral feeding is fat increased to 50%of total calories? B Schwartz’s: p.33
A. heart failure formulas C. renal failure formulas
B. pulmonary failure formulas D. calorie- defense formulas

127. the appendeceal flora remains constant throughout life with the exception of what bacteria, which A Schwartz’s: p.1129
appears only in adults?
A. Porphromonas gingivalis C. Peptostreptococci
B. Pseudomonas aeruginosa D. Streptococcus Anginosus

128. The most significant factor associated with both fetal and maternal deaths in pregnant patients with acute B Schwartz’s: p.1129
appendicitis is
A. formation of a phlegmon C. appendeceal tip near the uterus
B. appendeceal perforation D. retrocecal appendix

129. Which of the ff. is true of the superior mesenteric artery syndrome? C Schwartz’s: p.1027
A. it is often seen in obese individuals
B. it usually affects patients over 70 yrs. Old.
C. Compression is over the third portion of the duodenum
D. it is a surgical emergency
130. which of the following factors present in a entero-cutaneous fistula increases the possibility of closure of A Schwartz’s: p.1037
the tract?
A. Non-epithelialization of the tract C. Malnutrition
B. High output fistula D. Sepsis

131. what is the single most important factor in predicting burn related morbidity and mortality? A Schwartz’s 7th ed., p.228-232/Ans. A
A. size of the burn C. etiology of the burn
B. type of the burn D. associated medical condition

132. what type of shock is caused by the interference of the balance of vasolidator and vasoconstrictor D Schwartz’s 7th ed., p.113-115/Ans. D
influences to arterioles and venules?
A. cardiogenic shock C. septic shock
B. hypovolemic shock D. neurogenic shock

133. A 50 y/o female consulted to you and complains of fatigue, polydipsia, polyuria, nocturia, joint pains D Schwartz 7th ed., p. 1434-1439 / Ans. D
and constipation. Her laboratory exams show normal FBS, elevated serum calcium and elevated intact PTH.
What would be the most likely diagnosis?
A. NIDDM
B. Metastatic Breast Cancer
C. Hyperthyroidism
D. Hyperparathyroidism

134. A 35 y/o male presents with a long standing severe hypertension with associated muscle weakness, C Schwartz 7th ed., p. 1453-1458 / Ans. C
headache, polyuria and polydipsia. Blood exams revealed an elevated aldosterone level and elevated serum
potassium. What is the most likely diagnosis?
A. Cushing’s Syndrome
B. Pheochromocytoma
C. Conn’s Syndrome
D. Waterhouse-Friderichsen Syndrome

135. Which of the following is the most commonly injured organ in blunt abdominal injury? D Schwartz 7th ed., p. 187 / Ans. D
A. small intestine
B. large intestine
C. pancreas
D. spleen

136. In the initial management of an acutely and seriously injured patient, the first and most important D Schwartz 7th ed., p. 156 / Ans. D
emergency measure to be taken care of is:
A. splinting of fracture
B. control of bleeding
C. restoration of blood volume
D. ensuring adequate airway

137. Diagnostic peritoneal lavage (DPL) remains the most sensitive test available for determining the D Schwartz 7th ed., p. 167 / Ans. D
presence of intraabdominal injury. The result of DPL is considered grossly positive if:
A. 1 ml of blood is aspirated
B. 5 ml of blood is aspirated
C. 8 ml of blood is aspirated
D. 15 ml of blood is aspirated

138. Which of the following constitute an immediate threat to life because of inadequate ventilation? A Schwartz 7th ed., p. 156-157 / Ans. A
A. flail chest
B. hemothorax
C. stabbed wound involving the diaphragm
D. pulmonary hematoma

139. While undergoing exploratory laparotomy for blunt abdominal injury, the patients BP was noted to A Schwartz 7th ed., p. 193 / Ans. A
be 70 palpatory while bleeding was noted coming from the splenic hilum. Appropriate management for the
injured spleen should be:
A. splenectomy
B. splenorrhapy
C. debridement and repair
D. packing, immediate closure of abdominal lesion and volume replacement

140. During the initial resuscitation of a hypovolemic patient secondary to a vehicular accident, external B Schwartz 7th ed., p. 157 / Ans. B
bleeding is best controlled by:
A. suturing
B. direct finger pressure
C. application of tourniquet
D. blood transfusion

141. In penetrating neck injuries, zone II is referred to an area between: B Schwartz 7th ed., p. 165 / Ans. B
A. clavicle and cricoid cartilage
B. cricoid cartilage and angle of mandible
C. above the angle of the mandible
D. below the clavicle

142. In assessing the extent of hepatic injury following blunt injury to the abdomen, bleeding from the C Schwartz 7th ed., p. 188-189 / Ans. C
liver is best controlled by:
A. manual compression of the liver parenchyma
B. using figure of 8-suture
C. pringle maneuver
D. liver resection

143. Stable patients at risk of urethral injury, manifesting as presence of blood at the matus, necessitates B Schwartz 7th ed., p. 161-162 / Ans. B
an immediate:
A. urethral catheterization
B. urethrography
C. cystoscopy
D. KUB-IVP

144. Type of healing in which the wound is allowed to heal by granulation tissue formation and B Schwartz 7th ed., p. 234 / Ans. B
contraction:
A. primary intention
B. secondary intention
C. tertiary intention
D. quaternary intention

145. Partial thickness wound such as seen in superficial second degree burns heal by which of the following B Schwartz 8th ed., p. 228 / Ans. B
process:
A. wound contraction C. granulation tissue formation
B. epithelization D. maturation and remodelling

146. Equilibrium between collagen synthesis and collagen degradation occurred during the stage of: D Schwartz 8th ed., p. 228 / Ans. D
A. hemostasis and inflammation C. matrix synthesis
B. proliferative stage D. Remodelling

147. Delayed primary wound closure is indicated in which of the following type of wound: C Schwartz 7th ed., p. 264 / Ans. C
A. diabetic ulcer
B. decubitus ulcer
C. contaminated traumatic wound
D. surgical incision following simple appendectomy

148. Cytokines that are produced by one cell and affect and adjacent cell such as transforming growth factor B Schwartz 7th ed., p. 266 / Ans. B
beta (TGF-B) are called:
A. endocrine factors C. autocrine factors
B. paracrine factors D. intacrine factors

149. The major component of the extracellular matrix that provides strength, support and structure of all soft A Schwartz 7th ed., p. 270-271 / Ans. A
tissues, tendons, ligaments and bones is:
A. collagen C. fibronectin
B. elastin D. hyaluronic acid

150. The genetic disorder arising from mutations in the genes for the type I collagen causing increase B Schwartz 7th ed., p. 278 / Ans. B
propensity for the bones to break under minimal stress is:
A. ehlers-danlos syndrome C. marfan’s syndrome
B. osteogenesis imperfecta D. epidermolysis bullosa

151. Lesion of the gastrointestinal tract that result in complete regeneration and recovery: D Schwartz 7th ed., p. 280-281 / Ans. D
A. peptic ulcer disease C. radiation colitis
B. crohns disease D. gastric erosions

152. Skin lesions that extend beyond the boundaries of the original wound, do not regress with time and recur B Schwartz 7th ed., p. 281-282 / Ans. B
after excision:
A. hypertrophic scars C. diabetic ulcers
B. keloids D. decubitus ulcers

153. Kehr’s sign is a classic example of: D Schwartz 7th ed., p. 264 / Ans. D
A. unreferred visceral pain C. unreferred parietal pain
B. referred visceral pain D. referred parietal pain

154. The bleeding in Mallory-Weiss syndrome is secondary to: B Schwartz 7th ed., p. 1062-1063 / Ans. B
A. penetration of ulcer affecting the gastroduodenal artery
B. linear tears of the gastroesophageal junction
C. gastric erosions
D. hematobilia
155. The most common cause of intestinal obstruction for all age group combined is: C
A. strangulated hernis
B. volvulus
C. adhesive bands
D. neoplasm

156. The most obvious route of fluid and electrolyte loss in patients with intestinal obstruction from ileocecal C. vomiting
TB is through; Schwartz 7th ed., p. 1056/Ans. C
a. the edematous bowel wall
b. the intestinal lumen
c. vomiting
d. the peritoneal cavity

157. Occlusion of the blood supply to a segment of bowel in addition to obstruction of the lumen is referred to C. strangulated obstruction
as; Schwartz 7th ed., p. 1057/Ans. C
a. complete intestinal obstruction
b. closed-loop obstruction
c. strangulated obstructio
d. ileus

158. Which of the following diagnostic modalities consistently localizes the site of bleeding in cases of lower GI C. Angiography
bleeding; Schwartz 7th ed., p. 1066/Ans. C
a. Colonoscopy
b. CT scan
c. Angiography
d. Barium enema

159. The most common cause of massive lower GI bleeding is; B. diverticulosis
a. colonic malignancy Schwartz 7th ed., p. 1065/Ans. B
b. diverticulosis
c. adenomatous polyps
d. ulcerative colitis

160. A 50 y/o male with history of alcoholism was admitted because of history of progressive jaundice, C. Ultrasonography
associated with tea-colored urine, acholic stools, weight loss and mild epigastric pain. The simplest and most Schwartz 7th ed., p. 1071/Ans. C
non-invasve method in the diagnosis of this patient is;
a. percutaneous transhepatic cholangiography (PTC)
b. ERCP
c. Ultrasonography
d. HIDA scan

161. Relative to the ileum, the jejunum has; C. longer vasa recta
a. more fatty mesentery Schwartz 8th ed., p. 1018/Ans. C
b. less prominent plicae circularis
c. longer vasa recta
d. lesser in diameter

162. The most common malignant neoplasm of the small bowel is; A. Adenocarcinoma
a. Adenocarcinoma Schwartz 7th ed., p. 1242/Ans. A
b. carcinoids
c. sarcomas
d. lymphomas

163. The presence of a Meckels diverticulum in a hernial sac is called; C. littre's hernia
a. spingelian hernia Schwartz 7th ed., p. 1249/Ans. C.
b. femoral hernia
c. littre's hernia
d. richter's hernia

164. Rovsing's sign is elicited by; A. pain in the right lower quadrant when palpatory
a. pain in the right lower quadrant when palpatory pressure is exerted in the left lower quadrant pressure is exerted in the left lower quadrant
b. pain in the right lower quadrant when placatory pressure is exerted directly over the point of tenderness Schwartz 7th ed., p. 1385/Ans. A
c. flexion-extension of the right lower extremity
d. asking the patient to cough

165. In children with history of URTI 3 days prior to development of RLQ pain, the differential diagnosis most D. acute mesenteric adenitis
often confused with appendicitis is; Schwartz 7th ed., p. 1387/Ans. D
a. Meckel's diverticulitis
b. perforated peptic ulcer
c. acute gastroenteritis
d. acute mesenteric adenitis
166. The recommended treatment for patients with adenocarcinoma of the appendix is; B. right hemicolectomy
a. Appendectomy Schwartz 7th ed., p. 1392/Ans. B
b. right hemicolectomy
c. total colectomy
d. subtotal colectomy

167. The precipitating factor in secondary omental torsion is; D. foci of intraabdominal inflammation
a. bifid omentum Schwartz 7th ed., p. 1556/Ans. D
b. accessory omentum
c. obesity
d. foci of intraabdominal inflammation

168. The most common solid tumor of the omentum is; D. metastatic carcinoma
a. Lymphoma Schwartz 7th ed., p. 1558/Ans. D
b. liposarcoma
c. leiomyosarcoma
d. metastatic carcinoma

169. The most common anaerobic bacteria that is isolated in intraabdominal infection is: C. bacteriodes species
a. clostridium species Schwartz 7th ed., p, 1524-1525 / Ans. C
b. peptococcus species
c. bacteriodes species
d. Fusobacterium species

170. A 45 year old female came in because of right upper quadrant pain, colicky aggravated by fatty food intake B. acute cholecystitis
4 hours ptc. Physical exam showed a palpable mass at the right subcostal margin, non-movable, tender, Schwartz / Ans. B
midinspiratory arrest was illicited while the palpating hand was on the right subcostal area. The most likely
condition is/are:
a. acute appendicitis
b. acute cholecystitis
c. perforated peptic ulcer
d. acute ascending cholangitis

171. The laboratory examination of a 45 year old female came in with the following results: SGPT was elevated, C. acute calculous cholecystitis
direct bilirubin increased, alkaline phosphatase increased, protime prolonged. UTZ showed a hyperechoic mass Schwartz / Ans. C
at the neck of the gallbladder with posterior shadowing. The most likely diagnosis is:
a. acute appendicitis
b. acute infectious hepatitis
c. acute calculous cholecystitis
d. Ascending cholangitis

172. The treatment of choice for the above condition ( no.4 ) is: C. cholecystectomy
a. appendectomy Schwartz / Ans. C
b. medical management
c. cholecystectomy
d. ERCP

173. A patient s/p cholecystectomy and cbde with t-tube choledochostomy showed a distal cbd stone on t- C. choledococopy and basket extraction of the
tube cholangiography. Management would be: stone
a. daily irrigation with NSS Schwartz / Ans. C
b. ERCP
c. choledococopy and basket extraction of the stone
d. repeat explore lap and cbde

174. A 25 year old male came in with chronic draining lesion at the perianal area of 2 years duration. Physical B. posterior midline
exam showed a draining lesion about 4 cm from the anal verge at the 3:00 position. The internal opening is Schwartz / Ans. B
probably:
a. anterior midline
b. posterior midline
c. radially directly opposite
d. variable

175. 30 year old female came in because of painful perianal mass 2 days ptc. Physical exam showed a tender C. perianal abscess
mass at the right lateral position w/c precludes rectal exam with a bulging tender mass adjacent to it. The most Schwartz / Ans. C
likely condition is:
a. thrombosed external hemorhoids
b. prolapsed internal hemorhoid
c. perianal abscess
d. rectal polyp

176. A 50 year old male came in because of loss of weight about 30% of his previous body weight. Associated C. colonoscopy w/ biopsy
with on and off colicky abdominal pain. He also noticed that his stools have diminished in calibre just like a goat Schwartz / Ans. C
stool. The single most important examination would be:
a. fecalysis
b. Ba enema
c. colonoscopy w/ biopsy
d. CT scan

177. A 26 year old female came in with a palpable breast mass noted 2 weeks ptc. The mass is about 2.5 cm A. Fibroadenoma
well delineated border, movable, non-tender at the right upper outer quadrant, no familial history of ca. The Schwartz / Ans. A
most likely diagnosis would be:
a. Fibroadenoma
b. fibrocystic disease
c. breast ca
d. galactocoele

178. Seromas are fluid collection coming from the: C. Lymph


a. Plasma
b. Interstilium
c. Lymph
d. Capillary oozing

179. A patient was discharged 3 days after appendectomy (suppurative). After 5 days, he came back at the OPD B. Surgical site infection
for follow up complaining of tenderness at the operative site. The incision site was nonerythematous. What is
your diagnosis?
a. Hyperesthesia
b. Surgical site infection
c. Foreign body reaction to suture
d. Urinary Tract infection

180. A patient was operated on for acute cholecystitis. After 24h postop, the patient developed fever. What is D. Atelectasis
the most likely cause of this fever?
a. Cathether sepsis
b. Drug-related conditions
c. Aspiration pneumonia
d. Atelectasis

181. The basic protein in poor wound healing regardless of the underlying factor is: C. Low O2 tension
a. Poor local hemostasis
b. Anemia
c. Low O2 tension
d. Impaired inflammatory response

182. Ileus following abdominal surgery is expected to last for at least how many days? C. 5
a. 3
b. 4
c. 5
d. 6

183. For every degree rise in the temperature, the insensible loss in cm3 per day is approximately: D. 250
a. 100
b. 150
c. 200
d. 250

184. Which ion if altered determines the shift of fluid from one compartment to another? A. Sodium
a. Sodium
b. Potassium
c. Chloride
d. Bicarbonate

185. Which of the following cytokines may help control keloids and hypertrophic scars? d. TGB-B
a. PDGF
b. EGF
c.TNF
d. TGB-B

186. Which part of the GIT provides strength in the anastomosis? C. Submucosa
a. Serosa
b. Muscularis
c. Submucosa
d. Mucosa

187. Which amino acid provide energy source in the GIT? C. Glutamine
a. Alanine
b. Valine
c. Glutamine
d. Tryptopan

188. A 54 y/o male patient with 3rd degree burn approximately 40% TBSA developed abdominal pain on the 4th D. Renal colic
hospital day. What is the least likely cause of the abdominal pain?
a. Acalculous cholecystitis
b. Acute pancreatitis
c. Superior mesenteric artery syndrome
d. Renal colic

189. Convulsions if present during resuscitative phase of burn injury may be due to B. Hyponatremia
a. Hypoxemia
b. Hyponatremia
c. Infection
d. Hypokalemia

190. The offending organism in surgical site infection is: A. Staph aureus
a. Staph aureus
b. Pseudomonas
c. E coli
d. Bacteroides

191. Earliest manifestations of the catheter related complication following parenteral nutrition C. Glucose intolerance
a. Tachycardia
b. Fever
c. Glucose intolerance
d. Changes in sensorium

192. Which of the following statements is not a sound principle in the fluid and electrolyte therapy post B. Urine volume is replaced on a milliliter to
operatively? milliliter basis
a. Thorough evaluation of the pre and intra-op fluid status
b. Urine volume is replaced on a milliliter to milliliter basis
c. It is not necessary to give potassium with in the first 24 hours
d. Insensible water loss is considered in the fluid therapy

193. The average potassium (in meq/l) content of the bile per day is B. 10
a. 5
b. 10
c. 15
d. 18

194. Which cranial nerve is involved in the act of swallowing A. 11


a. 11
b. 10
c. 7
d. 12

195. Primary reason for staging esophageal cancer is A. Determine its resectability
a. Determine its resectability
b. Prognostication
c. To assess whether the procedure is for cure or palliation
d. Whether pre-op chemotherapy is indicated

196. Most common benign esophageal tumor is A. Leiomyomas


a. Leiomyomas
b. Fibromas
c. Myomas
d. Fibromyomas

197. Blood supply of the lesser curvature is B. Right gastric


a. Left gastric
b. Right gastric
c. Gastroepiploic
d. Splenic

198. Manifestation of acute gastric dilatation D. Hypotension


a. Pallor
b. Rapid respiration
c. Changes in sensorium
d. Hypotension

199. A patient underwent upper gastrointestinal endoscopy for gastric outlet obstruction. The endoscopist B. Polypoid
noted a lesion at the antrum and took biopsy. It turned out to be gastric cancer. What type of gastric cancer has
favorable prognosis?
a. Superficial spreading
b. Polypoid
c. Ulcerative
d. Linitis plastic

200. A patient diagnosed to have perforated peptic ulcer disease has his symptoms 8 hours prior to admission. D. omental patch
He was prepared for surgery. Which of the following best describes the required operation?
a. repair of perforation and selective vagotomy
b. repair of perforation and truncal vagotomy
c. suture of perforation
d. omental patch

201. After truncal vagotomy for peptic ulcer disease, the patient may develop the following except: B. megaloblastic anemia
a. gastric stasis
b. megaloblastic anemia
c. diarrhea
d. gallstone formation

202. Most common neoplasm of the stomach A. adenocarcinoma


a. adenocarcinoma
b. lymphoma
c. leiomyosarcoma
d. squamous cell carcinoma

203. A 77 y/o, M, chronic smoker on NSAIDs for arthritis presents with an acute abdomen and A. severe abdominal pain due to chronic gastritis
penumoperitoneum. What could be the most likely cause of this?
a. severe abdominal pain due to chronic gastritis
b. acute pancreatitis
c. acute cholecystitis with cholangitis
d. perforated peptic ulcer disease

204. Increased venous pressure, decreased pulse pressure and decreased heart sounds are pathognomonic of: C. cardiac tamponade
a. acute MI
b. pneumothorax
c. cardiac tamponade
d. aneurysm of the arch of aorta

205. “Bird’s beak” deformity is a radiologic sign of A. sigmoid volvulus


a. sigmoid volvulus
b. ileoascending intussuception
c. sigmoid carcinoma
d. cardiac tamponade

206. A 32 y/o M was brought to the ER because of blunt abdominal trauma due to fall. The patient had C. determine function of either kidneys
hematoma and abrasions on his RUQ and Rt flank. Your resident assessed that the patient has surgical
abdomen and needs immediate surgery. He requested one-shot intravenous pyelography en route to OR. The
reason for such,
a. find out extravasation of urine
b. localize site of obstruction
c. determine function of either kidneys
d. quantify volume of urine output

207. A 23 y/o female, medical student, was brought to the ER because of VA. Patient was GCS 13-14 with blunt D. does not rule out abdominal organ injury
abdominal trauma following a motor vehicle accident. Diagnostic peritoneal lavage was done and showed
negative results. The assessment of negative result signifies:
a. no hollow viscus injury
b. absent blood in peritoneal cavity
c. needle did not reach the peritoneal cavity
d. does not rule out abdominal organ injury

208. The most important factor in the management of contaminated wounds is C. adequate debridement
a. broad spectrum antiobiotics
b. anti-tetanus prophylaxis
c. adequate debridement
d. closure without tension

209. Whether it’s diverticulization or exclusion, the principle behind these procedures for the duodenal injuries B. to rest or isolate the injured duodenum
is to
a. prevent fistula formation
b. to rest or isolate the injured duodenum
c. prevent leakage
d. contain the inflammation at the RUQ
210. Sphincterotomy for anal fissures is usually done at the B. posterolateral
a. anteromidline
b. posterolateral
c. posteromidline
d. anterolateral

211. Surgical management of familial adenomatous polyposis depends on the A. number of polyps seen
a. number of polyps seen
b. size of polyps
c. histologic grading
d. staging

212. Elective colorectal cancer operation is classified as C. clean contaminated


a. clean
b. contaminated
c. clean contaminated
d. dirty

213. Which of the following is the most important prognostic determination of survival after treatment of D. histologic differentiation
colorectal cancer
a. lymph node involvement
b. transmural extension
c. tumor size
d. histologic differentiation

214. A 60 y/o male is admitted to the hospital after passing out large amount of maroon-colored stools. At the D. RBC tag scan
ER, he again passed out more bloody stools as well as clots. He is pale and tachycardic, NGT aspirate is bilious.
After resuscitation, which of the ff is the most appropriate initial test:
a. barium enema
b. rigid proctoscopy
c. colonoscopy
d. RBC tag scan

215. Which is the most important stimulus for triggering endocrine response to injury D. local wound factors
a. hypovolemia
b. afferent nerve stimulation from the injured area
c. tissue acidosis
d. local wound factors

216. Which is not true regarding gastrostomy tube feeding? D. preferably delivered in a constant flow
a. can be done endoscopically or open method
b. indicated for mentally obtunded patients
c. caloric requirement is based on patient’s needs
d. preferably delivered in a constant flow

217. Pellagra, dermatitis, glossitis, & peripheral paresthesias with spinal cord symptoms are due to deficiency A. Folic acid
of
a. Folic acid
b. Vitamin a
c. Pantothenic acid
d. Zinc

218. The primary source of energy following trauma is A. glycogen


a. glycogen
b. lipids
c. protein
d. carbohydrates

219. Eicosanoids are cell mediators derived from C. arachidonic acid


a. glycerol
b. cholesterol
c. arachidonic acid
d. glutamine

220. The reason why approximately one half of breast cancer are located at the upper outer quadrant is D. abundance of blood supply
a. its lymphatics
b. the predilection of cancers superiorly
c. the larger the volume of breast tissue
d. abundance of blood supply

221. Which structures demarcates the node levels in breast cancer: D. pectoralis minor
a. latissimus dorsi
b. pectoralis major
c. axillary artery
d. pectoralis minor

222. Carcinoma of the breast among the elderly presenting as a bulky, colloid tumor. A. Mucinous
a. Mucinous
b. Medullary
c. Adenoid
d. Apocrine

223. Management of inflammatory breast carcinoma with the best response A. Neoadjuvant chemo + MRM + irradiation
a. Neoadjuvant chemo + MRM + irradiation
b. Classical mastectomy + irradiation
c. Chemotherapy + irradiation
d. MRM + Hormonal manipulation

224. Which of the following structures which when encroached by cancer gives rise to the “orange-peel” D. Subdermal lymphatics
appearance of the breast?
a. Glands of Montgomery
b. Ductal ampulla
c. Lobules
d. Subdermal lymphatics

225. A 34 female patient came at the OPD with 2cm palpable mass, moveable, nontender. No axillary nodes A. Needle or open biopsy
noted. Patient has an elder sibling who died of breast cancer last 2 years ago. What will be your initial
diagnostic procedure?
a. Needle or open biopsy
b. Mammography
c. Open biopsy ultrasound of the breast
d. Magnetic resonance imaging

226. Collagen synthesis is at its peak at around how many days? C. 5-7
a. 1-2
b. 3-5
c. 5-7
d. 10-14

227. Formation of collagen fibers by cross-linking is enhanced by lysyl oxidase together with B. copper
a. magnesium
b. copper
c. zinc
d. chromium

228. Which of the following steroids useful in the treatment of scars and keloids B. Triamcinolone
a. Betamethasone
b. Triamcinolone
c. Prednisolone
d. Cortisone

229. Most common cause of hospital acquired infections are coming from the C. Urinary tract
a. Respiratory tract
b. Intraabdominal
c. Urinary tract
d. Blood

230. 45 male came in to the ER with a 3cm lacerated wound at the left leg after he was bitten by a pig. He has C. Debridement, leave wound open + tetanus
received tetanus toxoid and tetanus immunoglobulin 10 ½ years ago. What would be the appropriate immunoglobulin + toxoid+ antibiotics
management at the ER?
a. Toxoid +tetanus+ antibiotics
b. Tetanus immunoglobulin only
c. Debridement, leave wound open + tetanus immunoglobulin + toxoid+ antibiotics
d. Tt + ATS + Cloxacillin

231. Superior vena caval obstruction is commonly caused by B. Bronchogenic carcinoma


a. Fibrosing mediastinitis
b. Bronchogenic carcinoma
c. Caval thrombosis
d. Aortic aneurysm

232. The average length of the adult trachea is: D. 11 cm


a. 8 cm
b. 9 cm
c. 10 cm
d. 11 cm

233. Squamous cell carcinoma of the lung is commonly caused by A. hypercalcemia


a. hypercalcemia
b. hyperkalemia
c. hyperglycemia
d. hypomagnesemia

234. The initial diagnostic tool in assessing biliary ductal obstruction A. Alkaline phosphatase
a. Alkaline phosphatase
b. Ultrasonogrophy
c. ERCP
d. CT Scan

235. Progressive painless jaundice with positive “Courvoisier’s sign” suggest C. Pancreatic head or peri ampullary neoplasm
a. Choledocholithiasis
b. Hydrops of gall bladder
c. Pancreatic head or peri ampullary neoplasm
d. Choledochal cyst

236. The initial diagnostic procedure in patients presenting with lower GI bleed B. Proctosigmoidoscopy
a. NGT intubation
b. Proctosigmoidoscopy
c. Barium enema
d. Colonoscopy

237. The main parameter for determining the clinical stage of head and neck tumor is B. Lymph node involvement
a. Size of primary tumor
b. Lymph node involvement
c. Metastasis
d. Functional loss

238. Malignancy of the tongue usually occurs at the B. Midlateral


a. Tip
b. Midlateral
c. Central
d. Posterior

239. The second most frequent benign salivary gland neoplasm is C. Papillary cystadenoma lymphomatosum
a. Pleomorphic adenoma
b. Oxyphilic adenoma
c. Papillary cystadenoma lymphomatosum
d. Sialadenoma

240. The acidosis in hypovolemic shock is due to B. Lactic acid


a. Phosphate
b. Lactic acid
c. Carbonic acid
d. HCL

241. Substance associated with increased incidence of mesothelioma C. Asbestos


a. Smoking
b. Silica
c. Asbestos
d. Lead

242. Prostatic carcinoma usually metastasizes to the D. Bone


a. Liver
b. Kidneys
c. Bladder
d. Bone

243. The earliest visible evidence of neoplastic transformation is C. Dysplasia


a. Hyperplasia
b. Metaplasia
c. Dysplasia
d. Angiogenesis

244. Epstein-barr virus is associated with the following tumors, except D. Hepatocellular carcinoma
a. Burkitts lymphoma
b. Nasopharyngeal carcinoma
c. Hodgkin’s
d. Hepatocellular carcinoma
245. A normal to low CVP that does not rise with rapid administration of crystalloids indicates: A. Inadequate rate of infusion
a. Inadequate rate of infusion
b. Improper placement of CVP
c. Continuing hypovolemia
d. Improper choice of crytalloids

246. Recurrent laryngeal nerve is preserved in thyroid surgery. What is false with regards to this nerve: B. Injury to the ipsilateral nerve cause disability of
a. Injury to both nerves cause upper airway obstruction phonation
b. Injury to the ipsilateral nerve cause disability of phonation
c. It is a branch of cranial nerve x
d. 64% of the recurrent is seen along the tracheosophageal groove

247. Among the types of thyroid cancer, which one is the most aggressive? D. Anaplastic
a. Follicular
b. Papillary
c. Hurthle
d. Anaplastic

248. A 45yo female patient underwent total thyroidectomy for follicular ca. 24 H after surgery, the patient B. Ischemia to parathyroids
developed numbness around the lips and finger. What could have caused this?
a. Transection of recurrent nerve
b. Ischemia to parathyroids
c. Hematoma
d. Transection of superior laryngeal nerve

249. True regarding hernia: D. Repair of umbilical hernia is indicated in infants


a. Most common type of hernia in females is direct hernia with hernia > 2cm
b. Strangulation is more common in indirect than in femoral hernia
c. Tension-free repair is best achieved with bassini technique
d. Repair of umbilical hernia is indicated in infants with hernia > 2cm

250. A 23yo male patient came in at ER with irreducible inguinal hernia of >16 hours duration. Patient D. Schedule patient for surgery
complained of abdominal pain with BP of 100/70, CR 120, & RR 30. The appropriate management is:
a. Observe for progression of pain
b. Sedate patient and reduce the hernia
c. Apply ice pack over the hernia
d. Schedule patient for surgery

251. At the ER, a patient complained of renal colic. The following are the minimal laboratory evaluation you A. CBC and electrolytes
would request except:
a. CBC and electrolytes
b. IVP
c. Bun and creatinine
d. Urinalysis

252. In the medical treatment of BPH, which among the drugs acts by reducing the intraprostatic B. Finasteride
dihydrotestosterone levels without lowering the plasma testosterone level?
a. Doxazosin
b. Finasteride
c. Terazosin
d. Tamsulosin

253. A 48hour old baby boy was brought to the ER due to vomiting of bilious vomitus, abdominal distension, B. Ileal atresia
and failure to pass out meconium. What is the most likely cause?
a. Tracheo-esophageal atresia
b. Ileal atresia
c. Pyloric stenosis
d. Rectal mass

254. A 28 kg child with ruptured appendicitis has the following laboratory results: BP=90/60; PR=110; RR=25; C. actual Na deficit is 250 mEq
Na=125; Cl=95; K=3; ph=7.35; pCO2=35; pO2=100; HCO3=24. Which of the following is TRUE?
a. anion gap is increased
b. uncompensated metabolic acidosis is present
c. actual Na deficit is 250 mEq
d. the ideal intravenous fluid to infuse is D50.3% NaCl

255. Interpret the above ABG results: C. compensated metabolic acidosis


a. compensated respiratory alkalosis
b. uncompensated metabolic acidosis
c. compensated metabolic acidosis
d. uncompensated metabolic alkalosis
256. In the above patient, which of the following is TRUE? B. initial fluid resuscitation is 560 cc given as fast
a. plasma volume is 5,600 cc drip
b. initial fluid resuscitation is 560 cc given as fast drip
c. decrease in Na is primarily due to renal losses
d. decrease in Na is delusional in nature

257. In the same patient, which abnormality is he liable to manifest? A. increased intracranial pressure
a. increased intracranial pressure
b. dry sticky mucous membrane
c. shift of oxygen-hemoglobin dissociation curve to the left
d. ventricular arrhythmias

258. Extracellular deficit results in: B. increased haematocrit


a. increased sodium
b. increased hematocrit
c. decreased BUN
d. decreased creatinine

259. A 70-kg is admitted because of abdominal colic followed by vomiting of previously ingested food. Initial B. isotonic dehydration
assessment is an Intestinal Obstruction probably from adhesions. On examination, the patient has weak and
rapid pulse, depressed eyeballs, and dry tongue and mucous membranes.
The most likely fluid state of the patient is:
a. hypotonic dehydration
b. isotonic dehydration
c. hypertonic dehydration
d. hypernatremia

260. TRUE statements regarding fluid and electrolyte losses in this patient: B. can go into hypovolemic shock anytime
a. vomiting accounts for an insignificant amount of the volume loss
b. can go into hypovolemic shock anytime
c. sequestration of fluid in the urinary system accounts for majority of the losses
d. compensatory fluid shift from other compartments is sufficient to maintain homeostasis

261. Fluid replacement in this patient should begin with: C. crystalloids


a. blood
b. colloids
c. crystalloids
d. plasma

262. Which of the following is an expected cellular change/s in hypovolemic shock? D. increase in transmembrane potential
a. intracellular gain of Na+ and H2O
b. depletion of cellular ATP
c. extracellular increase of K+
d. increase in transmembrane potential

263. A patient has a blood pressure of 70/50 mmHg and a serum lactate level of 30-mg/100 ml (normal: 6 to B. hypovolemic shock
16). His cardiac output is 1.9 L/min, and his central venous pressure is 2 cmH2O. The most likely diagnosis is:
a. congestive heart failure
b. hypovolemic shock
c. cardiac tamponade
d. pulmonary embolus

264. During compensation for hypovolemic shock, blood flow to which of the following area/s is under C. small bowel
sympathetic nervous system control?
a. brain
b. heart
c. small bowel
d. liver

265. This hemodynamic monitor is valuable to differentiate a pericardial tamponade from hypovolemia: A. CVP
a. CVP
b. Arterial catheter
c. pulmonary catheter
d. capnograph

266. In a multiply injured patient with acute blood loss, adequate preload to the heart is maintained initially by A. increase in systematic vascular resistance
the:
a. increase in systematic vascular resistance
b. development of tachycardia
c. hormonal effects of angiotensin
d. hormonal effects of antidiuretic hormone
267. An example of neurogenic shock. B
A. Convulsion
B. Syncope
C. Carpopedal spasm
D. Involuntary fine tremors

268. Coagulation factors test by prothrombin time (PT) A


A. VII
B. XXII
C. VIII
D. XI

269. The initial hemostatic response to injury is C


A. Platelet formation
B. Hemolysis
C. Vascular constriction
D. Initiation of coagulation cascade

270. All are major events in the hemostatic process: B


A. Vascular dilatation
B. Platelet plug formation
C. Fibrin deposition
D. Hemolysis

271. The following are suggestive of DIC: B


A. Platelet counts of 250,000/cu. Mm
B. Increased fibrin split products
C. Normal fibrinogen level
D. Hypofribrinogenemia

272. Injury to the intima of blood vessels exposes the subendothelial collagen which initiates: B
A. Fibrinolysis
B. Platelet aggregation
C. Release of cyclic AMP
D. Vasoconstriction

273. A 77-year old man is schedules to undergo sigmoid colectomy. He denies any history of prolonged D
bleeding. Preoperative evaluation of hemostasis should include:
A. No screening tests
B. Prothrombin time (PT) and partial thromboplastin time (PTT)
C. Platelet count, blood smear, and PTT
D. Platelet count, PT, and PTT

274. The total caloric requirement of a malnourished patient is calculated using his: A
A. Actual weight
B. 75% of ideal body weight
C. 50% of ideal body weight
D. Ideal body weight

275. In prescribing the nutrition therapy, micronutrients are given at what % of RDA: A
A. 100
B. 80
C. 60
D. 50

276. Nitrogen excretion is less in A


A. Starvation
B. Sepsis
C. Severe burns
D. Elective surgery

277. At the initial stages of shock it is normally expected to have a A


A. Normal blood pressure
B. Hypertension
C. Cold extremities
D. Hypothermia

278. A 38-year old male had abdominal exploration for multiple gunshot wounds. He is febrile T 38.5 C on the A
2nd post-op day. HR is 95 bpm, BP 100/80 mm Hg, WBC count 13,000. The patient is hooked to a
ventilator. The patient is developing what condition:
A. SIRS
B. Sepsis
C. Severe sepsis
D. Septic shock

279. On the 5th day post-op the patient in addition to the above findings now shows erythema and draining pus B
from the abdominal incision site. The patient now is developing:
A. SIRS
B. Sepsis
C. Severe sepsis
D. Septic shock

280. On the 10th day post-op, the patient is persistently, febrile with increase in WBC count (16,000) and this D
time with oliguria that is unresponsive to fluid resuscitation. The patient now is developing:
A. SIRS
B. Sepsis
C. Severe sepsis
D. Septic shock

281. The above patient was given pressor agents to improve his condition, despite this he remains oliguric. The D
patient now has:
A. SIRS
B. Sepsis
C. Severe sepsis
D. Septic shock

282. Immediately after mastectomy your patient asks if any of the following promotes wound healing, which A
one will you advise your patient:
A. Vitamin C
B. Zinc supplementation
C. Vitamin B
D. Carbohydrate rich foods

283. Severe cases of hidradenitis suppurativa in the groin area are best managed by excision of the involved A
area and:
A. Closure by secondary intention
B. Delayed primary closure
C. Primary closure
D. Partial thickness skin grafts

284. An elderly cancer patient fell and sustained a deep lacerated wound over the right forehead about 9 cm
length. If wound infection later develops in this patient. The major cause of impaired wound healing is:
A. Anemia
B. Local wound infection
C. DM
D. All of the above

285. What technique of wound closure is recommended in a patient with ruptured appendix with spreading B
peritonitis?
A. Primary
B. Delayed primary
C. Secondary
D. Tertiary

286. If the lacerated wound is sutured closed, this healing is known as: A
A. Primary
B. Secondary
C. Tertiary
D. Delayed primary

287. Wound healing in this this patient may be impaired because of his: A
A. Age
B. Depth of the wound
C. Type of wound
D. Cancer

288. Infection in the above wound is partly controlled by the presence of what cells: A
A. Macrophages
B. Fibroblast
C. Endothelial cells
D. T lymphocytes

289. Important activities of macrophages during wound healing A


A. Wound debridement
B. Arginine synthesis
C. Activation of coagulation cascade
D. Fibrin clot

290. The ABC’s of resuscitation includes the following: A


A. Establish adequate airway
B. Bleeding should be controlled by applying tourniquet
C. Circulation should be maintained by repaid infusion of blood
D. Oxygen should be delivered if necessary

291. A 25 year old male is brought to the ER after he sustained a stab wound to the 5th intercostal space. PE: C
conscious, stretcher-borne, vitals are as follows: blood pressure 90/50 mm Hg, pulse 110/min and
respiratory rate 30/min. Which of the following statements is true:
A. Primary survey and initial resuscitation should be instituted one after the other
B. A complete detailed physical exam should be performed so as not to miss any injury
C. Large-bore IV lines should inserted
D. Blood for CBC and x-matching should be drawn

292. An 8 year old boy has a closed angulated deformity of his right forearm. VS: BP=99mmHg systolic and D
PR=110. Identify the most emergent problem of the victim. Write any of the following:
A. Airway or Cervical Spine
B. Breathing
C. Circulation
D. Disability or Neurologic Injury

293. A 7 year old boy is gasping for breath. He can speak clearly, but complains of chest pain and an inability to B
get his breath. He is becoming increasingly anxious. VS: BP=110 mmHg systolic and PR=110. Identify the
most emergent problem of the victim. Write any of the following:
A. Airway or Cervical Spine
B. Breathing
C. Circulation
D. Disability or Neurologic Injury

294. A 25 year old man is unconscious, withdraws from painful stimuli, eyes are closed but they open in B
response to pain, and he is making unintelligible sounds. His pupils are equal, and both react sluggishly to
light. His Glasgow coma scale score is.
A. 5
B. 8
C. 11
D. 14

295. After control of this patient’s airway is achieved, the first diagnostic study to obtain is: C
A. x-rays of the skull
B. CT scan of the head
C. x-rays of the cervical spine
D. carotid angiography

296. A 19 year old woman presents with a non-bleeding stab wound, 1 cm long, in the anterior triangle of her D
neck, 3 cm above her left clavicle. Examination of the neck reveals that the wound has penetrated the
platysma muscle and that the left carotid pulse is normal; there is no hematoma or bruit. Appropriate
management of this patient:
A. immediate tracheostomy
B. nasogastric intubation
C. chest x-rays
D. formal neck exploration under general anesthesia

297. Following an automobile accident, a 30 year old woman is discovered to have posterior pelvic fracture. C
Hypotension and tachycardia respond marginally to volume replacement. Once it is evident that her
major problem is free intraperitoneal bleeding and a pelvic hematom in association with the fracture,
appropriate management would be:
A. apply antishock trousers and exploratory laparatomy
B. arterial embolizataion of the pelvic vessels
C. exploratory laparatomy and possible bilateral internal iliac artery ligation
D. external pelvic fixation to stabilize the pelvis

298. Large amount of free intraperitoneal blood is seen in a patient with penetrating abdominal injury after an C
abdominal ultrasound, the next best step is to:
A. apply antishock trousers and exploratory laparatomy
B. arterial embolizataion of the pelvic vessels
C. exploratory laparatomy
D. external pelvic fixation to stabilize the pelvis

299. A 6 year old is having episodes of watery diarrhea for the past three days. She is lethargic, irritable and C
restless. The heartbeat is rapid but the pulses are weak. The child should:
A. stay home and call the paediatrician for advice
B. stay home and have a cold batch
C. proceed to the nearest hospital emergency room
D. get a glass of cold fruit juice to drink

300. Over- enthusiastic fluid resuscitation should be avoided in this child because of D
A. potential liver failure
B. Acute myocardial infarction
C. venous thrombosis and rupture
D. potential cerebral injury

301. A study on ruptured appendicitis will be conducted in a tertiary university hospital that includes all age A
groups and all genders on the rate of SSI between primary vs. delayed primary closure. What is the
possible confounder in this study?
A. all age groups
B. ruptured appendicitis
C. SSI
D. primary vs. delayed primary

302. The most common type of inguinal hernia in both males and females is: A
A. indirect
B. direct
C. femoral
D. mixed

303. The Cooper’s ligament repair for direct or femoral hernia is popularized by: B
A. Bassini
B. McVay
C. Halsted
D. Ferguson

304. A 22 year old female consulted a physician because of a 2x2x3 cms, movable well circumscribed, non- C
tender mass, noted accidentally while taking a bath. Physical examination was consistent with the
findings above. Most like diagnosis is:
A. Breast Ca
B. Fibrocystic change
C. Fibroadenoma
D. Phylloides tumor

305. The next thing to do on the above patient is: A


A. reassurance and observation
B. ask for mammogram
C. Partial mastectomy
D. Hormonal reatment

306. Ivory an 18 year old female consulted because of breast tenderness becoming more severe just before C
menses. PE showed asymmetric nodularities on the upper outer quadrant of both breasts with no
dominant mass. Menstrual cycle is monthly and regular. Most likely diagnosis is:
A. Sclerosing adenosis
B. Normal premenstrual tension
C. Fibrocystic change
D. Breast Ca

307. Sarah Jane a 35 year old female consulted because of an intermittent bloody nipple discharge on the right D
breast. PPE-no mass noted on the right breat. Most likely diagnosis is:
A. papillary cancer
B. mammary duct ectasia
C. bleeding galactocoele
D. intraductalpapilloma

308. Chemotherapy in breast carcinoma is: B


A. given to those with at least 4 positive xillary metastasis
B. usually a combination of multiple agents
C. free of any side effects
D. used to control local recurrence

309. Breast malignancy that is known to have the highest incidence of bilaterality and multicentricity: D
A. Paget’s disease of the breast
B. lobular carcinoma
C. Papillary carcinoma
D. Medullary carcinoma
310. The next appropriate thing to do in managing the above case will be: A
A. Local wound exploration
B. CT scan
C. Immediate surgery
D. Triple angiography of the abdomen

311. Fifteen minutes later the BP became 60 palpatory, PR-120/min accompanied by abdominal distension. D
You will recommend:
A. CT scan
B. proceed with local wound exploration
C. MRI
D. Immediate surgery

312. The most commonly injured intraabdominal organ following a blunt abdominal trauma: B
A. Pancreas
B. Spleen
C. small intestine
D. colon

313. A 55 y/o male, smoker underwent FNAB for a 4 cm preauricular mass. Results showed (+) for malignant C
cells, patient is most probably suffering from:
A. Pleomorphic adenoma
B. Follicular carcinoma
C. mucoepidermoid carcinoma
D. papillary carcinoma

314. The appropriate treatment for a benight parotid neoplasm is: B


A. “shelling out” of the tumor
B. superficial parotidectomy
C. total parotidectomy
D. radiation therapy

315. Most common histology of head-neck cancers is C


A. Adenocarcinoma
B. Papillary carcinoma
C. squamous cell carcinoma
D. Adenosquamous carcinoma

316. The most common causative agent being implicated in head-neck cancers is B
A. UV radiation
B. Tobacco
C. nitrates
D. alcohol

317. The site of injury corresponds to which zone of the neck? A


A. Zone 1
B. Zone 2
C. Zone 3
D. Zone 4

318. The diagnosis of the above is: B


A. Primary hyperparathyroidism
B. Secondary Hyperparathyroidism
C. Tertiary Hyperparathyroidism
D. Parathyroid Carcinoma

319. The course of papillary thyroid cancer is best described by which of the following statements? C
A. Metastases are rare; local growth is rapid
B. Local invasion and metastases almost never occur
C. Metastases frequently occur to cervical lymph nodes, distant metastases and local invasion are rare
D. Rapid widespread metastatic involvement of the liver, lungs and bone results in a 10% 5 yr survival
rate

320. The most important prognosticating factor in thyroid cancer is: B


A. tumor size
B. patient’s age
C. lymph node metastases
D. Histologic grade

321. 43 years old female present with diffuse enlargement of the thyroid gland accompanied by palpitation, C
heat intolerance and weight loss. She is most probably suffering from
A. Frey’s syndrome
B. Hypothyroidism
C. hypocalcemia
D. hypokalemic acidosis
322. A 50-year old male has just undergone a total thyroidectomy for follicular cancer of the thyroid. On the C
third hour post-op, he experienced circumoral numbness. He is probably suffering from
A. Frey’s syndrome
B. hypothyroidism
C. hypocalcemia
D. hypokalemic acidosis

A 16 year old high school student came to you with a complaint that her friends took notice of an anterior neck C
enlargement since she started on her weight reduction program. You were indecisive as to whether there really
was a thyromegaly. The rest of the P.E. were normal

323. Which would be most helpful in determining whether there really is a thyromegalyor none:
A. a second opinion from an endocrinologist
B. MRI of the neck
C. Thyroid Scanning
D. Serum TSH, T3 and T4

324. A 72 year-old male consults because of change in bowel habits for the last two weeks. On rectal exam, a B
hard irregular mass was noted at 6 cm from the anal verge. The most likely diagnosis here is:
A. Rectal carcinoid
B. Rectal carcninoma
C. Rectal polyp
D. Anal canal carcinoma

325. The most appropriate initial diagnostic procedure to be done is B


A. ultrasound of the abdomen
B. Proctosigmoidoscopy
C. Chest x-ray
D. CT scan of the pelvis

326. Majority of peptic ulcer patients are successfully managed medically or conservatively. Which of the B
following complications is the most common indication for surgical intervention?
A. Perforation
B. Massive GI bleeding
C. Gastric outlet obstruction
D. Malignant degeneration

327. A 48 year old company manager who has been taking proton pump inhibitor irregularly for the past 2 A
years because of on and off epigastric discomfort presents in the emergency room with hematemesis and
melena. This patient should be:
A. Aggressively resuscitated with fluids and measures to localize site of bleeding should be done in
preparation for possible surgery
B. Explored immediately because bleeding is an indication for surgery
C. Give massive doses of anti-ulcer medications
D. Watchful waiting for 24 to 48 hours

328. A 43 year old male patient with on and off epigastric pain during the past 15 months complains of sudden D
severe epigastric pain later on becoming generalized. An upright chest file taken at the ER shows a strip of
lucency underneath the right hemidiaphragm. This patient has:
A. Penetrating peptic ulcer
B. Acute pancreatitis
C. Perforated diverticulitis
D. Perforated peptic ulcer

329. Above patient is best managed by: B


A. Aggressive medical treatment for PUD
B. Laparotomy after resuscitation/IV antibiotics
C. Fluid resuscitation, nasogastric decompression and somatostatin
D. Emergency ERCP

330. A 30 y/o male who usually has dripping fresh blood after defecation suddenly had anal pain and a D
smooth, tender 1cm diameter rounded mass coming out of the anus. This is most likely:
A. peri-anal abscess
B. internal hmorrhoids 3rd degree
C. anal wart
D. thrombosed internal haemorrhoids

331. A 65 y/o male was referred for surgical consult because of abdominal pain and distention. Findings in the D
scout film of the abdomen that will be highly suggestive of intestinal obstruction
A. air in distal rectum
B. pneumatosis intestinales
C. luscency beneath the right hemidiaphragm
D. step ladder abnormality
332. A 75 y/o female is being worked up for paralytic ileus. The electrolyte that is usually below normal is: A
A. potassium
B. sodium
C. calcium
D. magnesium

333. A significant and independent influence on the prognosis of a patient who had an esophageal resection A
for cancer is:
A. tumor presentation of the esophageal wall
B. cell type
C. degree of cellular dfferentation
D. location of the tumor in the esophagus

334. Hepatic resection should be consider for localized metastatic spread from which of the following primary A
site?
A. colorectal
B. stomach
C. lungs
D. breast

335. The standard of treatment for patient with biliary colic and cholecystolithiasis by ultrasound is A
A. Cholecystectomy
B. Cholecystostomy
C. Stone dissolution
D. Endoscopic Stone removal

336. In an infected obstructed common bile duct producing cholangitis, which of the following is the best C
option for drainage in a very poor risk patient?
A. cholecystostomy
B. operative CBD exploration and T tube choledochostomy
C. ERCP and stenting
D. MRICP

337. CT scan finding that is highly indicative of infected pancreatic necrosis: D


A. edema
B. dilated pancreatic duct
C. calcifications
D. gas bubbles in the retroperitoneum

338. When gallstone ileus occurs, obstruction is most frequent in: C


A. duodenum
B. jejunum
C. distal ileum
D. sigmoid colon

339. The most common cause of acute cholecystitis is: A


A. cystic duct obstruction
B. E. coli infection
C. gall bladder polyp near the neck
D. multiple gallstones

340. The strongest evidence of presence of gallstones by ultrasound is: D


A. edema and thickening of gallbladder wall
B. contracted gallbladder
C. dilated extrahepatic bile ducts
D. High level echoes with posterior acoustic shadowing

A 45 y/o jaundiced patient who has been diagnosed to have gallbladder and CBD stones by ultrasound 6 C
months ago is brought to the ER because of abdominal pain, chills, high grade fever and hypotension. PE: icteric
sclera, abdomen slightly tender with guarding RUQ.

341. The patient most likely is having which of the fllowing:


A. acute cholecystitis
B. empyema of the GB
C. acute suppurative obstructive cholangitis
D. ruptured GB

342. The problem of patient in the previous number is best managed by: C
A. administration of oral antibiotics
B. careful observation and follow up
C. surgical/endoscopic decompression of biliary tree after resuscitation and antibiotics
D. give systemic antibiotics and steroids
343. The most likely diagnosis in a 70 y/o male presenting with significant weight loss accompanied by C
progressive jaundice, anorexia, pruritus, and tea colored urine. PE showed a palpable nontender
gallbladder.
A. hepatoma
B. gallbladder CA
C. pancreatic head CA
D. gallbladder empyema

344. The severity of pancreatitis, particularly in those patients who are not improving after 24hours of medical
management can be assessed using:
A. angiography
B. scout film of the abdomen
C. dynamic CT scan
D. MRICP

345. Which of the following clinical clues is a reliable symptom of arteriosclerosis obliterans? D
A. poor hair growth
B. deformed toenails
C. cold hands and feet
D. intermittent claudication

346. What is the most common symptom of acute aortic dissection? B


A. syncope
B. severe pain
C. hemiparesis
D. congestive heart failure

347. This is the BEST procedure for a patient with acute head injury secondary to a vehicular accident: A
A. CT scan
B. angiography
C. plain skull x-rays
D. electroencephalogram

348. Abrasion wounds on the face are best treated with: A


A. wound cleansing and debridement with normal saline
B. split thickness skin grafting
C. laser surgery
D. AOTA are correct

349. Essential elements in the survival of a skin graft: C


A. antibiotics
B. dry dressing
C. vascularity
D. nerve supply

350. Most common cause of UTI in children: A


A. E. coli
B. S. saprophyticus
C. Klebsiella, Enterobacter
D. Proteus sp.

351. A massively bleeding posterior duoedenal ulcer, which of the following is most likely involved: B
A. left gastric artery
B. gastroduodenal artery
C. short gastric artery
D. left gastroepiploic artery

352. Which of the following would inhibit parietal cell acid secretions? D
A. histamine
B. acetylcholine
C. gastrin
D. prostaglandins

353. The most common symptom associated with a Meckel’s diverticulum is: C
A. crampy abdominal pain
B. periumbilical pain
C. blood per rectum
D. watery diarrhea

354. The typical visual field deficit caused by a pituitary adenoma with suprasellar extension is: C
A. bitemporal hemianopsia
B. homonymous hemianopsia
C. superior quadrantanopsia
D. Inferior quadrantanopsia

355. The evaluation of a comatose patient with a head injury begins with: C
A. evaluation of cardiovascular system
B. evaluation of papillary reflexes
C. establishment of an airway
D. plain film of the skull

356. The radial nerve is at greatest risk for injury with which fracture: B
A. fracture of the surgical neck of the humerus
B. fracture of the shaft of the humerus
C. supracondylar fracture of the humerus
D. olecranon fracture

357. An early sign of compartment syndrome in the hand includes: A


A. pain with passive stretch at the digits
B. absent radial pulse
C. motor paralysis
D. swelling of the digits

358. The most common site of involvement for skeletal TB is the: D


A. tibia
B. femur
C. pelvis
D. spin

359. A 65 y/o woman with a history of chronic constipation is transferred from a nursing home because of C
abdominal pain and marked abdominal distension. On examination, her abdomen is found to be
distended and tender in the LLQ. The most likely diagnosis is:
A. appendicitis
B. CA of the colon
C. volvulus of the sigmoid colon
D. small bowel obstruction

360. A 68-year old man presents with crampy abdominal pain and distention vomiting. Findings on PE are A
positive for healed abdominal scars. X-ray reveals multiple gas fluid levels. The WBC count is 12,000. The
most likely diagnosis is.
A. Small Bowel obstruction due to adhesions
B. A Hernia
C. Appendicitis
D. Gallstones and Ascites

361. A 56- year old man has suffered from intermittent claudication for 5 years. He has recently developed B
abdominal pain that is made worse by eating. He has history of a 15-lb weight loss. The most likely
diagnosis is:
A. Chronic Cholecystitis
B. Chronic Intestinal Ischemia
C. Peptic Ulcer
D. Abdominal Aortic aneurysm

362. A 28-year old man with a history of emotional disturbance enters the hospital with a history of weight A
loss and regurgitation of food. Regurgitation is worse when he lies down. The most likely diagnosis is:
A. Hiatal Hernia
B. Cancer of the esophagus
C. Duodenal Ulcer
D. Achalasia

363. A 38-year old man with a history of fever associated with abdominal pain of several weekws duration B
presents now with a sudden onset of explosive abdominal pain and vomiting. Flat plate x-ray reveals air
under the diaphragm. CT scan shows mesenteric lymphadenopathy and splenomegaly. The most likely
diagnosis is:
A. Tubercolosis enteritis
B. Typhoid enteritis
C. Primitive peritonitis
D. Ulcerative colitis

364. Splenectomy is often indicated in the management of: A


A. Hereditary Spherocytosis
B. Hereditary neurofibromatosis
C. Aplastic anemia
D. Hashimoto’s disease
365. A painless distended gallbladder palpable on PE of jaundice patient is strongly suggestive of: D
A. Empyema of the gallbladder
B. Mirizzi’s Syndrome
C. Gallstone impacted on ampulla
D. Pancreatic Carcinoma

366. Which of the ff. suggest unresectability of a left upper lobe lung cancer: B
A. Hemoptysis
B. Malignant pleural effusion
C. A cough specimen with positive sputum cytology
D. Clubbing and blueness of fingers

367. Which of the ff. studies is contraindicated in a drowsy patient with papilledema whom one suspects of B
having closed head trauma:
A. Carotid arteriography
B. Lumbar puncture
C. CT scna
D. Echoencephalography

368. The appropriate antibiotic for a patient with a cellulitis of a leg due to streptococcus is: A
A. Penicillin
B. Erythromycin
C. 3rd generations cephalosporin
D. Cloxacillin

369. Massive bleeding from the lower GI tract (beyond the ligament of Treitz) is most often due to: A
A. Diverticulosis
B. Meckel’s diverticulum
C. Diverticulus
D. Colonic Carcinoma

400. A 54-year old male has been complaining of recurrent hypogastric pain for almost 6 months. For almost 3 D
weeks prior to consult, he had noticed that he is passinf air during micturition. The most common cause
of the above condition is:
A. Carcinoma of the bladder
B. Tubercolosis of the urinary tract
C. CA of the sigmoid colon
D. Diverticulitis of the colon

401. A 42-year old woman has been complainging of intermittent gross hematuria. On urinalysis, she has D
significant pyuria but without bacilluria. The condition is most commonly due to:
A. Urethritis
B. Bladder stone
C. Perinephric abscess
D. Tubercolosis of the kidney

402. A 25-year old male had episodes of severe vomiting due to gastric outlet obstruction. He appears pale and C
dehydrated. The most likely metabolic anomality the patient would have is:
A. Hypochloremic acidosis
B. Hyponatremic acidosis
C. Hypochloremic alkalosis
D. Hypernatremic alkalosis

403. A 1 week old infant presents wuth moderate respiratory distress and tympany in only one hemithorax. C
The most likely diagnosis is:
A. Spontaneous pneumothorax
B. Eventration of the diaphragm
C. Bochdalek hernia
D. Atelectasis

404. A little boy aspirated a peanut. He is cyanotic. On expiration, his mediatinum shifts to the right. The B
peanut most likely is lodged in the:
A. Right main stem bronchus
B. Left main stem bronchus
C. Trachea
D. Esophagus

405. A 40-year old man is hit by a car and sustains an injury to the pelvis. Which of the ff. is most indicative of a B
urethral injury:
A. Hematuria
B. High-riding prostrate on rectal examination
C. Oliguria
D. Scrotal ecchymosis
406. A 64-year old man has mild upper abdominal pain. In contrast CT-scan, a 5 cm lesion in the left lobe of the D
liver enhances and then decreases over a 10-minute period from without to within. The most likely lesion
is a:
A. Hepatic adenoma
B. Amoebic abscess
C. Fatty infiltration
D. Cavernous hemangioma

407. A 28-year old male has had a nonseminomatous testicular cancer treated. In followind this patient for B
possible recurrent tumor, the most useful serum marker would be:
A. Carcinoembryonic antigen
B. Alpha fetoprotein
C. PSA
D. Alkaline phosphatase

408. The most common cause of esophageal rupture or perforation: A


A. Endoscopic injury
B. Blunt chest trauma
C. Boarhaave’s syndrome
D. Carcinoma

409. The superior mesenteric artery communicates with the celiac artery via the: A
A. Pancreatico-duodenal artery
B. Splenic artery
C. Hepatic aretey
D. Dorsal pancreatic artery

410. The most common offending organism in pyogenic osteomyelitis is: B


A. Eschericia coli
B. Staphylococcus aureus
C. P. aeruginosa
D. Enterococcus

411. In patients who developed a documented episode of deep vein thrombosis, the most frequent and C
significant long term sequel is:
A. Claudication
B. Recurrent foot infection
C. Development of stasis ulcer
D. Pulmonary embolism

412. In an arterial stenosis, the most critical factor is: B


A. The length of the stenosis
B. The cross-sectional area of the stenosis
C. The length and area are equally important
D. Whether the stenosis is smooth or has an irregular countour

413. A 60-year old amn with a history of atrial fibrillation is found to have a cyanotic cold right lower extremity. D
The embolous most probably originated from:
A. An atherosclerotic plaque
B. Deep vein thrombosis
C. Lungs
D. Heart

414. Which of the ff. elements is not a component of venous thrombogenesis: A


A. Incompetent valves in perforating vein
B. Disruption of endothelial intima
C. Venostasis
D. Hypercoagulability

415. In a patient suffering from an acute arterial embolic phenomenon. If the ischemia is not relieved, which of A
these ff. tissues is the first to be irreversibly damaged?
A. Muscle
B. Nerve
C. Fat
D. Synovial Membrane

416. Which of the ff. clinical manifestation may suggest tentorial herniation: C
A. Contralateral mydriasis
B. Decorticate rigidity
C. Contralateral hemiparesis
D. Ipsilateral hemianopsia
417. The most common complication of lung abscess is: A
A. Empyema
B. Pneumothorax
C. Brochopleural fistula
D. Osteomyelitis of the ribs

418. The most common posterior-superior sulcus chest tumor in a 6-month old child is: A
A. Neuroblastoma
B. Teratoma
C. Cystoic hygroma
D. Ganglioneuroma

419. The most common cause of spontaneous pneumothorax is: B


A. Tubercolosis
B. Emphysematous blebs
C. Pneumonia
D. Bronchial rupture

420. What substance is released by macrophages in order to activate T helper/inducer (CD4+) lymphocytes? A
A. Interleukin 1
B. Interleukin 2
C. Interleukin 3
D. Interleukin 4

421. A 27-yr old hypertensive, diabetic woman is admitted for cadaveric renal transplantation. She is blood B
type B and has had four transfusions of packed cells over the preceding 6 months. Which of the following
factors would preclude transplantation?
A. Donor blood type O
B. Positive crossmatch
C. Two-antigen HLA match with donor
D. Blood pressure of 180/100 mmHg

422. The primary mechanism of action of cyclosporine A is inhibition of: D


A. Macrophage function
B. Antibody production
C. Cytotoxic T cell effectiveness
D. IL-2 production

423. Which of the following malignancy exhibits a slow growing, late metastasizing characteristic? B
A. Squamous cell carcinoma
B. Basal cell carcinoma
C. Melanoma
D. Basosquamous carcinoma

424. Following the Goodsall-Salmon’s rule in fistula in ano, an anteriorly located opening less than 3 cm from A
the anal opening follows:
A. Straight tract
B. Curve tract
C. No tract
D. S-shaped tract

425. One of the following statements is correct regarding hyponatremia: C


A. Serum sodium determination would give normal result
B. Hypoglycemia may be a cause
C. Management could either be water restriction or replacement depending on the cause
D. Hyperactive deep tendon reflex is a manifestation

426. A 60 kg lean male would normally have this amount of body water A
A. 36 liters
B. 24 liters
C. 30 liters
D. 48 liters

427. A practical but useful clinical parameter that can be used to assess the adequacy of volume replacement B
of patients suffering from hypovolemia is:
A. Central venous pressure measurement
B. Urine output
C. Blood pressure
D. Pulse Rate

428. A 42-year old man who has been on prolonged total parenteral nutrition (TPN) administration was noted D
to have scaly, hyperpigmented lesions over the acral surface of elbows and knees and alopecia. The most
likely cause of the condition is:
A. Essential fatty acid deficiency
B. Excess glucose calories
C. Hypomagnesemia
D. Zinc deficiency

429. A 40-year old man is found to have severe metabolic acidosis with a high anion gap. The most likely cause
is:
A. Diarrhea
B. Renal tubular acidosis
C. Ureterosigmoidostomy
D. Methanol ingestion

430. A 60-year old female, most mastectomy for breast cancer, presents with headache backache and frequent A
vomiting. She is extremely thirsty and stuporous. The test most likely to identify the cause is:
A. Serum calcium determination
B. Serum sodium determination
C. Serum potassium determination
D. Serum glucose determination

431. The Trendelenburg’s (head down) position is beneficial in the initial management of which type of shock: D
A. Hypovolemic shock
B. Cardiogenic shock
C. Septic Shock
D. Neurogenic Shock

432. The most common form of burn is: A


A. scald burns
B. flame burns
C. chemical burn
D. electrical burn

433. A 20 year old man has lymphadenopathy behind and inferior to his right ear. Biopsy shows the lesion to B
be alymphosarcoma. The most likely site of the primary tumor is:
A. Floor of the mouth
B. Nasopharynx
C. Buccal mucosa
D. Mediastinum

434. The most frequent organism in highest density that can be isolated from contamination following colon D
perforation are:
A. Gram positive aerobes
B. Gram negative aerobes
C. Gram positive anaerobes
D. Gram negative anaerobes

435. The most clinically significant early physiologic abnormality in post traumatic pulmonary insufficiency is: A
A. Increased physiologic shunt
B. Increased dead space
C. Cardiogenic pulmonary edema
D. Pulmonary fibrosis

436. Overall bone mass increases up to what age after which there is an overall decrease in bone mass: D
A. 10-15 years of age
B. 15-20 years of age
C. 20-25 years of age
D. 30-35 years of age

437. After multiple myeloma, the most common primary malignant bone tumor is: A
A. Osteosarcoma
B. Ewing’s sarcoma
C. Chodrosarcoma
D. Fibrosarcoma

355. The best treatment for septic arthritis of the hip is: C
A. Repeated daily aspiration
B. Arthroscopic drainage
C. Surgical drainage/arthrotomy with antibiotic therapy
D. Antibiotic therapy only

356. The most common form of inflammatory arthritis: B


A. Osteoarthritis
B. Rheumatoid arthritis
C. Septic arthritis
D. Psoriatic arthritis

357. The gold standard of treatment of femoral shaft fractures in adults is: C
A. Closed reduction and hip spica application
B. Open reduction and rearmed intramedullary nailing
C. Closed locked antegrade intramedullary nailing
D. Plating

358. Initial treatment of Talpesequinovarus (clubfoot) is: A


A. Serial casting immediately after birth/time of diagnosis
B. Immediate operative treatment
C. Dennis Brown splint
D. Pavlik harness

359. A unique feature of immature bone is the capacity to undergo plastic deformation without breaking and D
to sustain an incomplete fracture also called:
A. Nightstick fracture
B. Monteggia fracture
C. Galleazzi fracture
D. Greenstick fracture

360. Inflammation of the tendons in the first dorsal compartment containing the Abductor pollicis brevis and C
extensor pollicis brevis tendons is called:
A. Carpal Tunnel syndrome
B. Trigger finger
C. De Quervain’s Tenosynovitis
D. Lateral epicondylitis

Potrebbero piacerti anche